All Exams  >   Bank Exams  >   IBPS PO Prelims & Mains Preparation  >   All Questions

All questions of Reading Comprehension for Bank Exams Exam

1 Crore+ students have signed up on EduRev. Have you? Download the App

Once surrounded and protected by vast wilderness, many of the national parks are adversely affected by activities outside their boundaries. The National Park Organic Act established the national park system and empowered the Secretary of the Interior to manage activities within the parks. Conditions outside park boundaries are not subject to regulation by the Park Service unless they involve the direct use of park resources.  
Several approaches to protecting the national parks from external degradation have been proposed, such as one focusing on enacting federal legislation granting the National Park Service broader powers over lands adjacent to the national parks. Legislation addressing external threats to the national parks twice passed the House of Representatives but died without action in the Senate. Also brought to the table as a possible remedy is giving the states bordering the parks a significant and meaningful role in developing federal park management policy.  
Because the livelihood of many citizens is linked to the management of national parks, local politicians often encourage state involvement in federal planning. But, state legislatures have not always addressed the fundamental policy issues of whether states should protect park wildlife.  
Timber harvesting, ranching and energy exploration compete with wildlife within the local ecosystem. Priorities among different land uses are not generally established by current legislation. Additionally, often no mechanism exists to coordinate planning by the state environmental regulatory agencies. These factors limit the impact of legislation aimed at protecting park wildlife and the larger park ecosystem
Even if these deficiencies can be overcome, state participation must be consistent with existing federal legislation. States lack jurisdiction within national parks themselves, and therefore state solutions cannot reach activities inside the parks, thus limiting state action to the land adjacent to the national parks. Under the supremacy clause, federal laws and regulations supersede state action if state law conflicts with federal legislation, if Congress precludes local regulation, or if federal regulation is so pervasive that no room remains for state control. Assuming that federal regulations leave open the possibility of state control, state participation in policy making must be harmonized with existing federal legislation.
The residents of states bordering national parks are affected by park management policies. They in turn affect the success of those policies. This interrelationship must be considered in responding to the external threats problem. Local participation is necessary in deciding how to protect park wildlife. Local interests should not, however, dictate national policy, nor should they be used as a pretext to ignore the threats to park regions. 
Direction: Read the above Paragraph and answer the follownig Quetions
Q. What is the main purpose of the author in writing the passage? 
  • a)
    argue that rampant timber harvesting is degrading national parks  
  • b)
    describe a plan of action to resolve an issue 
  • c)
    discuss different approaches to dealing with a problem 
  • d)
    suggest that local participation is necessary to solve the problem described 
  • e)
    to assert that national parks are adversely affected by activities outside their boundaries 
Correct answer is option 'C'. Can you explain this answer?

Nandita Tiwari answered
Mapping the Passage:
¶1 describes a problem facing national parks: negative effects from the land
surrounding them.
¶2 describes one approach to dealing with the problem: federal legislation, which
failed.
¶3 and 4 describe a second approach: giving power to states to cooperate with
adjacent national parks, and describe the problems with it.
¶5 argues that state participation must be tied to federal regulations.
¶6 argues that any solution requires a national response with elements of local
participation.
If you have mapped the passage correctly you will notice that most of the passage
discusses the different approaches that can be taken to solve the problem of
degradation of national parks. C matches best with this.

Practice Quiz or MCQ (Multiple Choice Questions) with solutions are available for Practice, which would help you prepare for "Comprehension" under Verbal Aptitude. You can practice these practice quizzes as per your speed and improvise the topic. The same topic is covered under various competitive examinations like - CAT, GMAT, Bank PO, SSC and other competitive examinations
Q.
 
He saw nothing, he had no knife or sharp instrument, the grating of the window was of iron and he had too often assured himself of its solidity. His furniture consisted of a bed, a chair, a table, a pail, and a jug. The bed had iron clamps, but they were screwed to the wall and it would have required a screwdriver to take them off.
Dantes had but one resource which was to break the jug and with one of the sharp fragments attack the wall. He left the jug fall on the floor and it broke in pieces. He concealed two or three of the sharpest fragments in his bed, leaving the rest on the floor. The breaking of the jug was too natural an accident to excite suspicion, and next morning gaoler went grumblingly to fetch another, without giving himself the trouble to remove the fragments. Dantes heard joyfully the key grate in the lock as guard departed.
Dantes was in
  • a)
    a hostel
  • b)
    a dining room
  • c)
    an army barracks
  • d)
    a prison
Correct answer is option 'D'. Can you explain this answer?

Meera Rana answered
In the passage, it has been talked about the bed had iron clamps, but they were screwed to the wall and it would have required a screwdriver to take them off.
i.e. attached bed means jailer and the word Gaoler has the same meaning. 
So the answer is prison.
 

The passage below is accompanied by four questions. Based on the passage, choose the best answer for each question.
The Second Hand September campaign, led by Oxfam . . . seeks to encourage shopping at local organisations and charities as alternatives to fast fashion brands such as Primark and Boohoo in the name of saving our planet. As innocent as mindless scrolling through online shops may seem, such consumers are unintentionally—or perhaps even knowingly —contributing to an industry that uses more energy than aviation. . . .
Brits buy more garments than any other country in Europe, so it comes as no shock that many of those clothes end up in UK landfills each year: 300,000 tonnes of them, to be exact. This waste of clothing is destructive to our planet, releasing greenhouse gasses as clothes are burnt as well as bleeding toxins and dyes into the surrounding soil and water. As ecologist Chelsea Rochman bluntly put it, “The mismanagement of our waste has even come back to haunt us on our dinner plate.”
It’s not surprising, then, that people are scrambling for a solution, the most common of which is second-hand shopping. Retailers selling consigned clothing are currently expanding at a rapid rate . . . If everyone bought just one used item in a year, it would save 449 million lbs of waste, equivalent to the weight of 1 million Polar bears. “Thrifting” has increasingly become a trendy practice. London is home to many second-hand, or more commonly coined ‘vintage’, shops across the city from Bayswater to Brixton.
So you’re cool and you care about the planet; you’ve killed two birds with one stone. But do people simply purchase a second-hand item, flash it on Instagram with #vintage and call it a day without considering whether what they are doing is actually effective?
According to a study commissioned by Patagonia, for instance, older clothes shed more microfibres. These can end up in our rivers and seas after just one wash due to the worn material, thus contributing to microfibre pollution. To break it down, the amount of microfibres released by laundering 100,000 fleece jackets is equivalent to as many as 11,900 plastic grocery bags, and up to 40 per cent of that ends up in our oceans. . . . So where does this leave second-hand consumers? [They would be well advised to buy] high-quality items that shed less and last longer [as this] combats both microfibre pollution and excess garments ending up in landfills. . . .
Luxury brands would rather not circulate their latest season stock around the globe to be sold at a cheaper price, which is why companies like ThredUP, a US fashion resale marketplace, have not yet caught on in the UK. There will always be a market for consignment but there is also a whole generation of people who have been taught that only buying new products is the norm; second-hand luxury goods are not in their psyche. Ben Whitaker, director at Liquidation Firm B-Stock, told Prospect that unless recycling becomes cost-effective and filters into mass production, with the right technology to partner it, “high-end retailers would rather put brand before sustainability.”
Based on the passage, we can infer that the opposite of fast fashion, ‘slow fashion’, would most likely refer to clothes that:
  • a)
    Are of high quality and long lasting.
  • b)
    Do not bleed toxins and dyes.
  • c)
    Are sold by genuine vintage stores.
  • d)
    Do not shed microfibres.
Correct answer is option 'A'. Can you explain this answer?

Moumita Sen answered
Explanation:
There are several key points in the passage that can help us infer the opposite of fast fashion, which is 'slow fashion':
- High quality and long lasting: Slow fashion refers to clothing that is made to be durable, timeless, and of high quality. This is in direct contrast to fast fashion, which produces cheap, trendy clothes that are meant to be disposable after a few wears.
- Focus on sustainability: Slow fashion aims to reduce the environmental impact of the fashion industry by promoting practices such as recycling, upcycling, and using eco-friendly materials. This is evident in the passage's emphasis on buying second-hand items, which helps combat waste and pollution.
- Avoiding microfiber shedding: Slow fashion garments are designed to shed less microfibers, which can contribute to pollution in oceans and rivers. This contrasts with fast fashion items, which are often made from synthetic materials that shed microfibers easily.
Therefore, based on the information provided in the passage, we can infer that the opposite of fast fashion, 'slow fashion', would most likely refer to clothes that are of high quality, long lasting, sustainable, and do not shed microfibers.

In the decades following World War II, American business had undisputed control of the world economy, producing goods of such high quality and low cost that foreign corporations were unable to compete. But in the mid-1960s the United States began to lose its advantage and by the 1980s American corporations lagged behind the competition in many industries. In the computer chip industry, for example, American corporations had lost most of both domestic and foreign markets by the early 1980s
The first analysts to examine the decline of American business blamed the U.S. government. They argued that stringent governmental restrictions on the behaviour of American corporations, combined with the wholehearted support given to foreign firms by their governments, created and environment in which American products could not compete. Later analysts blamed predatory corporate raiders who bought corporations, not to make them more competitive in the face of foreign competition, but rather to sell off the most lucrative divisions for huge profits.  
Still later analysts blamed the American workforce, citing labour demands and poor productivity as the reasons American corporations have been unable to compete with Japanese and European firms. Finally, a few analysts even censured American consumers for their unpatriotic purchases of foreign goods. The blame actually lies with corporate management, which has made serious errors based on misconceptions about what it takes to be successful in the marketplace.
These missteps involve labour costs, production choices, and growth strategies.  
Even though labour costs typically account for less than 15% of a product‘s total cost, management has been quick to blame the costs of workers‘ wages for driving up prices, making American goods uncompetitive. As a result of attempts to minimize the cost of wages, American corporations have had trouble recruiting and retaining skilled workers.
 The emphasis on cost minimization has also led to another blunder: an over-concentration on high technology products. Many foreign firms began by specializing in the mass production and sale of low technology products, gaining valuable experience and earning tremendous profits. Later, these corporations were able to break into high technology markets without much trouble; they simply applied their previous manufacturing experience and ample financial resources to the production of higher quality goods. American business has consistently ignored this very sensible approach.  
The recent rash of corporate mergers and acquisitions in the U.S. has not helped the situation either. While American firms have neglected long-range planning and production, preferring instead to reap fast profits through mergers and acquisitions, foreign firms have been quick to exploit opportunities to ensure their domination over future markets by investing in the streamlining and modernization of their facilities.
Direction: Read the above Paragraph and answer the follownig Quetions
Q.  Which of the following would most weaken the author‘s argument about the over-concentration on high technology products?  
  • a)
    Producing low tech products is not as profitable as producing high tech products. 
  • b)
    Manufacturing high tech products is a completely different process than manufacturing low tech goods.  
  • c)
    Most of the low tech products purchased by Americans are made by foreign firms.  
  • d)
    Most of the high tech products purchased by Americans are made by foreign firms.
  • e)
    Most of the high tech products purchased by Americans are made by American firms.
Correct answer is option 'B'. Can you explain this answer?


Mapping the Passage
¶1 outlines the decline of American business
¶s2 and 3 list reasons that analysts have given for the decline and introduce the author‘s own theory for American business problems: incompetent management.
¶4 lists management‘s problems with labour.
¶5 explains the problem with America's fixation on high-tech products.
¶6 uses mergers to show that corporations lack long-range planning.
Paraphrase the author‘s argument about high technology: it‘s better to start out with low-tech, get experience, and then ramp up to high-tech. Search the answer choices for something that would contradict this. (B) clearly does; if the processes are completely different, why start with low-tech.

Practice Quiz or MCQ (Multiple Choice Questions) with solutions are available for Practice, which would help you prepare for "Comprehension" under Verbal Aptitude. You can practice these practice quizzes as per your speed and improvise the topic. The same topic is covered under various competitive examinations like - CAT, GMAT, Bank PO, SSC and other competitive examinations.
Q.
In the world today we make health and end in itself. We have forgotten that health is really means to enable a person to do his work and do it well. a lot of modern medicine and this includes many patients as well as many physicians pays very little attention to health but very much attention to those who imagine that they are ill. Our great concern with health is shown by the medical columns in newspapers. the health articles in popular magazines and the popularity of television programmes and all those books on medicine. We talk about health all the time. Yet for the most part the only result is more people with imaginary illness. The healthy man should not be wasting time talking about health: he should be using health for work. The work does the work that good health possible.
Modern medicine is primarily concerned with
  • a)
    promotion of good health
  • b)
    people suffering from imaginary illness
  • c)
    people suffering from real illness
  • d)
    increased efficiency in work
Correct answer is option 'B'. Can you explain this answer?

Vikas Kapoor answered
‘A lot of modern medicine, and this includes many patients as well as many physicians, pays very little attention to health but a lot of attention to those who imagine that they are ill.'
 

In the decades following World War II, American business had undisputed control of the world economy, producing goods of such high quality and low cost that foreign corporations were unable to compete. But in the mid-1960s the United States began to lose its advantage and by the 1980s American corporations lagged behind the competition in many industries. In the computer chip industry, for example, American corporations had lost most of both domestic and foreign markets by the early 1980s
The first analysts to examine the decline of American business blamed the U.S. government. They argued that stringent governmental restrictions on the behaviour of American corporations, combined with the wholehearted support given to foreign firms by their governments, created and environment in which American products could not compete. Later analysts blamed predatory corporate raiders who bought corporations, not to make them more competitive in the face of foreign competition, but rather to sell off the most lucrative divisions for huge profits.
 Still later analysts blamed the American workforce, citing labour demands and poor productivity as the reasons American corporations have been unable to compete with Japanese and European firms. Finally, a few analysts even censured American consumers for their unpatriotic purchases of foreign goods. The blame actually lies with corporate management, which has made serious errors based on misconceptions about what it takes to be successful in the marketplace. These missteps involve labour costs, production choices, and growth strategies.
 Even though labour costs typically account for less than 15% of a product‘s total cost, management has been quick to blame the costs of workers‘ wages for driving up prices, making American goods uncompetitive. As a result of attempts to minimize the cost of wages, American corporations have had trouble recruiting and retaining skilled workers.
The emphasis on cost minimization has also led to another blunder: an over-concentration on high technology products. Many foreign firms began by specializing in the mass production and sale of low technology products, gaining valuable experience and earning tremendous profits. Later, these corporations were able to break into high technology markets without much trouble; they simply applied their previous manufacturing experience and ample financial resources to the production of higher quality goods. American business has consistently ignored this very sensible approach.  
The recent rash of corporate mergers and acquisitions in the U.S. has not helped the situation either. While American firms have neglected long-range planning and production, preferring instead to reap fast profits through mergers and acquisitions, foreign firms have been quick to exploit opportunities to ensure their domination over future markets by investing in the streamlining and modernization of their facilities. 
Direction: Read the above Paragraph and answer the follownig Quetions
Q. With which of the following general statements would the author most likely NOT agree?
  • a)
    American business has been hurt by the inability to plan for the longterm.  
  • b)
    Cutting production costs always leads to increased competitiveness.  
  • c)
    American consumers are not the prime cause of the decline of American business.  
  • d)
    Initial analysis of the decline of American business yielded only partially accurate conclusions.                  
  • e)
    Mergers and Acquisitions have not helped improve the situation 
Correct answer is option 'B'. Can you explain this answer?

Maulik Das answered

Mapping the Passage
¶1 outlines the decline of American business
¶s2 and 3 list reasons that analysts have given for the decline and introduce the author‘s own theory for American business problems: incompetent management.
¶4 lists management‘s problems with labour.
¶5 explains the problem with America's fixation on high-tech products.
¶6 uses mergers to show that corporations lack long-range planning.
An inference question; make sure that you‘re clear on the main points of the author‘s argument. Remember that the author will agree with four, but will disagree with the correct answer. The three wrong answers could be easily eliminated, leading to (B). However, you can also reason that since management has suffered by cutting labour costs, cost-cutting doesn‘t always result in lowered prices.

In the decades following World War II, American business had undisputed control of the world economy, producing goods of such high quality and low cost that foreign corporations were unable to compete. But in the mid-1960s the United States began to lose its advantage and by the 1980s American corporations lagged behind the competition in many industries. In the computer chip industry, for example, American corporations had lost most of both domestic and foreign markets by the early 1980s.
 The first analysts to examine the decline of American business blamed the U.S. government. They argued that stringent governmental restrictions on the behaviour of American corporations, combined with the wholehearted support given to foreign firms by their governments, created and environment in which American products could not compete. Later analysts blamed predatory corporate raiders who bought corporations, not to make them more competitive in the face of foreign competition, but rather to sell off the most lucrative divisions for huge profits.
Still later analysts blamed the American workforce, citing labour demands and poor productivity as the reasons American corporations have been unable to compete with Japanese and European firms. Finally, a few analysts even censured American consumers for their unpatriotic purchases of foreign goods. The blame actually lies with corporate management, which has made serious errors based on misconceptions about what it takes to be successful in the marketplace.
These missteps involve labour costs, production choices, and growth strategies.  
Even though labour costs typically account for less than 15% of a product‘s total cost, management has been quick to blame the costs of workers‘ wages for driving up prices, making American goods uncompetitive. As a result of attempts to minimize the cost of wages, American corporations have had trouble recruiting and retaining skilled workers.  
The emphasis on cost minimization has also led to another blunder: an over-concentration on high technology products.
Many foreign firms began by specializing in the mass production and sale of low technology products, gaining valuable experience and earning tremendous profits. Later, these corporations were able to break into high technology markets without much trouble; they simply applied their previous manufacturing experience and ample financial resources to the production of higher quality goods.
American business has consistently ignored this very sensible approach.  The recent rash of corporate mergers and acquisitions in the U.S. has not helped the situation either. While American firms have neglected long-range planning and production, preferring instead to reap fast profits through mergers and acquisitions, foreign firms have been quick to exploit opportunities to ensure their domination over future markets by investing in the streamlining and modernization of their facilities.
Direction: Read the above Paragraph and answer the follownig Quetions
Q. The passage suggests that compared to Japanese workers, American workers are often considered:  
  • a)
     more content and more efficient.  
  • b)
    more content but less efficient.  
  • c)
    less content and less efficient.  
  • d)
    less content but more efficient.  
  • e)
    lazy and less hard working 
Correct answer is option 'C'. Can you explain this answer?

Gargi Kulkarni answered

Mapping the Passage
¶1 outlines the decline of American business
¶s2 and 3 list reasons that analysts have given for the decline and introduce the author‘s own theory for American business problems: incompetent management.
¶4 lists management‘s problems with labour.
¶5 explains the problem with America's fixation on high-tech products.
¶6 uses mergers to show that corporations lack long-range planning.
A quick scan of the answer choices shows that you have to compare the workers of the two nations on two criteria: contentedness and efficiency. Search for a part of the passage that touches on this. ¶3 is the only one that cites Japan, and mentions that analysts consider American workers less productive and less content. (C) it is.

Suspicious as they are of American intentions, and bolstered by court rulings that seem to give them license to seek out and publish any and all government secrets, the media‘s distrust of our government, combined with their limited understanding of the world at large, damages our ability to design and conduct good policy in ways that the media rarely imagine.
 The leak through which sensitive information flows from the government to the press is detrimental to policy in so far as it almost completely precludes the possibility of serious discussion. The fear that anything they say, even in what is construed as a private forum, may appear in print, makes many people, whether our own government officials or the leaders of foreign countries, unwilling to speak their minds. 
Must we be content with the restriction of our leaders‘ policy discussions to a handful of people who trust each other, thus limiting the richness and variety of ideas that could be brought forward through a larger group because of the nearly endemic nature of this problem? It is vitally important for the leaders of the United States to know the real state of affairs internationally, and this can occur only if foreign leaders feel free to speak their minds to our diplomats.  
Until recently, it looked as if the media had convinced the public that journalists were more reliable than the government; however, this may be changing. With the passage of time, the media have lost lustre. They—having grown large and powerful—provoke the same public skepticism that other large institutions in the society do. A series of media scandals has contributed to this. Many Americans have concluded that the media are no more credible than the government, and public opinion surveys reflect much ambivalence about the press.  
While leaks are generally defended by media officials on the grounds of the public‘s ―right to know,‖ in reality they are part of the Washington political power game, as well as part of the policy process. The "leaker" may be currying favour with the media, or may be planting information to influence policy. In the first case, he is helping himself by enhancing the prestige of a journalist; in the second, he is using the media as a stage for his preferred policies. In either instance, it closes the circle: the leak begins with a political motive, is advanced by a politicized media, and continues because of politics. Although some of the journalists think they are doing the work, they are more often than not instruments of the process, not prime movers. The media must be held accountable for their activities, just like every other significant institution in our society, and the media must be forced to earn the public‘s trust. 
Direction: Read the above Paragraph and answer the follownig Quetions
Q. What is the main idea of the passage?  
  • a)
    to argue that the media is acting against the national interests.
  • b)
    to convince that journalists are attempting to enhance their own prestige.                
  • c)
    to discuss the negative effects that media ―leaks‖ have on foreign policy and the media‘s credibility.              
  • d)
    to criticise politicians for being dishonest in public.        
  • e)
    to suggest that the media needs to be regulated more strongly and effectively. 
Correct answer is option 'C'. Can you explain this answer?

Milan Ghosh answered
Mapping the Passage
¶1 argues that the media‘s suspicion of government and lack of knowledge about the world harm government policy.
¶s2 and 3 introduce the concept of the ―leak and explain why it‘s bad for foreign policy.
¶4 states that the media was trusted by the public until recently, but are now met with skepticism.
¶5 argues that leaks are usually part of a power grab and that the media is a pawn in the game.
C is the most consistent with our passage summary above.

Those who opine lose their impunity when the circumstances in which they pontificate are such that generate from their expression a positive instigation of some mischievous act. An opinion that corn dealers are starvers of the poor, or that owning private property is robbery, ought to be unmolested when simply circulated through the press, but may justly incur punishment when delivered orally to an excited mob assembled before the house of a corn dealer, or when handed about among the same mob in the form of a placard. Acts, of whatever kind, which without justifiable cause do harm to others, may be, and in the more important cases are absolutely required to be, controlled by the unfavourable sentiments, and, when needful, by the active interference of mankind. The liberty of the individual must be thus far limited; he must not make himself a nuisance to other people. But if he refrains from molesting others in matters that concern them, and merely acts according to his own inclination and judgment in matters which concern himself he should be allowed, without molestation, to carry his opinions into practice at his own cost. As it is useful that while mankind are imperfect there should be different opinions, so it is that there should be different experiments of living, that free scope should be given to varieties of character, short of injury to others, and that the worth of different modes of life should be proved practically, when anyone thinks fit to try them. Where not the person‘s own character but the traditions and customs of other people are the rule of conduct, there is wanting one of the principal ingredients of individual and social progress.  
It would be absurd to pretend that people ought to live as if nothing whatever had been known in the world before they came into it; as if experience had as yet done nothing toward showing that one mode of existence, or of conduct, is preferable to another. Nobody denies that people should be so taught and trained in youth as to know and benefit by the ascertained results of human experience. But it is the privilege and proper condition of a human being, arrived at the maturity of his faculties, to use and interpret experience in his own way. It is for him to find out what part of recorded experience is properly applicable to his own circumstances and character. The traditions and customs of other people are, to a certain extent, evidence of what their experience has taught them—presumptive evidence, and as such, have a claim to his deference—but, in the first place, their experience may be too narrow, or they may have not interpreted it rightly. Secondly, their interpretation of experience may be correct, but unsuited to him. Customs are made for customary circumstances and customary characters, and his circumstances or his character may be uncustomary. Thirdly, though the customs be both good as customs and suitable to him, yet to conform to custom merely as custom does not educate him or develop in him any of the qualities which are the distinctive endowments of a human being. He gains no practice either in discerning or desiring what is best. 
Directions: Read the above paragraph and answer the following
Q.The author holds that one should not necessarily defer to the traditions and customs of other people. The author supports his position by arguing that:  
I. traditions and customs are usually the result of misinterpreted experiences.  
II. customs are based on experiences in the past, which are different from modern experiences.
III. customs can stifle one‘s individual development. 
  • a)
    II only  
  • b)
    III only  
  • c)
    I and III only  
  • d)
    II and III only  
  • e)
    None of the above 
Correct answer is option 'B'. Can you explain this answer?

Moumita Rane answered
Mapping the Passage:
¶1 (first half) introduces the limits that should be placed on expression of opinions.
¶1 (second half) argues that differences of opinions are necessary.
¶2 offers three reasons why individual opinions are necessary to personal growth.
Use your reasoning from the last question to help yourself with this one. Remember to eliminate answer choices as you either select or eliminate Roman numerals. RN III is in three choices, so look there first. The author argues that customs aren‘t always good, and therefore can stifle growth. RN III is correct. Eliminate (A). Next to RN II: it also contradicts the author‘s argument that customs can be useful. By default, the answer must be (B), but check RN I to be sure. The author defends the general usefulness of customs. While sometimes experiences are misinterpreted, there‘s nothing to indicate that the usually are, which knocks RN I out immediately. RN II.

While many points are worth making in an evaluation of the single sixyear presidential term, one of the most telling points against the single term has not been advanced. This kind of constitutional limitation on elections is generally a product of systems with weak or non-existent political parties.
Since there is no party continuity or corporate party integrity in such systems, there is no basis for putting trust in the desire for re-election as a safeguard against mismanagement in the executive branch. Better under those conditions to operate on the basis of negative assumptions against incumbents. I do not know if the earliest proposal for a single, nonrepeatable term was made in the 1820s because that was a period of severely weak political parties. But I do feel confident that this is a major reason, if not the only reason, that such a proposal has been popular since the 1940s.
Though the association of the non-repeatable election with weak political parties is not in itself an argument against the limitation, the fallout from this association does contribute significantly to the negative argument. Single-term limitations are strongly associated with corruption. In any weak party system, including the presidential system, the onus of making deals and compromises, both shady and honourable, rests heavily upon individual candidates. Without some semblance of corporate integrity in a party, individual candidates have few opportunities to amortize their obligations across the spectrum of elective and appointive jobs and policy proposals. The deals tend to be personalized and the payoffs come home to roost accordingly.
If that situation is already endemic in conditions of weak or nonexistent parties, adding to it the limitation against re-election means that candidates and officials, already prevented from amortizing their deals across space, are also unable to amortize their obligations temporally. This makes for a highly beleaguered situation. The single six-year term for presidents is an effort to compensate for the absence of a viable party system, but it is a compensation ultimately paid for by further weakening the party system itself.
 Observers, especially foreign observers, have often noted that one source of weakness in American political parties is the certainty of election every two or four years, not only because any artificial limitation on elections is a violation of democratic principles but also because when elections are set in a certain and unchangeable cycle, political parties do not have to remain alert but can disappear into inactivity until a known point prior to the next election. To rigidify matters by going beyond the determinacy of the electoral cycle to add an absolute rule of one term would hang still another millstone around the neck of already doddering political parties.  
Directions: Read the above paragraph and answer the following:
Q. Which of the following, if true, would most weaken the author‘s claim about single-term political systems?  
  • a)
    The discovery that foreign observers like this system          
  • b)
    The discovery that most politicians are honest          
  • c)
    The discovery that Americans dislike this system            
  • d)
    The discovery that parliamentary systems are more democratic              
  • e)
     The discovery that politicians favour such a system 
Correct answer is option 'B'. Can you explain this answer?

Mapping the Passage
¶1 introduces the idea of a single presidential term.
¶2 argues that the single term is usually associated with countries with weak political parties and therefore popular when political parties are weak.
¶s3 and 4 argue that single-term systems encourage corruption.
¶5 argues that the single-term system is designed to make up for weak parties, but ends up making parties even weaker.
What is the author‘s main argument for avoiding the single-term system? It leads to corruption. Looking for something that would weaken this argument turns up (B), which contradicts the author‘s main reason for avoiding the single-term system.

Suspicious as they are of American intentions, and bolstered by court rulings that seem to give them license to seek out and publish any and all government secrets, the media‘s distrust of our government, combined with their limited understanding of the world at large, damages our ability to design and conduct good policy in ways that the media rarely imagine.
 The leak through which sensitive information flows from the government to the press is detrimental to policy in so far as it almost completely precludes the possibility of serious discussion. The fear that anything they say, even in what is construed as a private forum, may appear in print, makes many people, whether our own government officials or the leaders of foreign countries, unwilling to speak their minds. 
Must we be content with the restriction of our leaders‘ policy discussions to a handful of people who trust each other, thus limiting the richness and variety of ideas that could be brought forward through a larger group because of the nearly endemic nature of this problem? It is vitally important for the leaders of the United States to know the real state of affairs internationally, and this can occur only if foreign leaders feel free to speak their minds to our diplomats.  
Until recently, it looked as if the media had convinced the public that journalists were more reliable than the government; however, this may be changing. With the passage of time, the media have lost lustre. They—having grown large and powerful—provoke the same public skepticism that other large institutions in the society do. A series of media scandals has contributed to this. Many Americans have concluded that the media are no more credible than the government, and public opinion surveys reflect much ambivalence about the press.
While leaks are generally defended by media officials on the grounds of the public‘s ―right to know,‖ in reality they are part of the Washington political power game, as well as part of the policy process. The "leaker" may be currying favour with the media, or may be planting information to influence policy. In the first case, he is helping himself by enhancing the prestige of a journalist; in the second, he is using the media as a stage for his preferred policies. In either instance, it closes the circle: the leak begins with a political motive, is advanced by a politicized media, and continues because of politics. Although some of the journalists think they are doing the work, they are more often than not instruments of the process, not prime movers. The media must be held accountable for their activities, just like every other significant institution in our society, and the media must be forced to earn the public‘s trust. 
Direction: Read the above Paragraph and answer the follownig Quetions
Q. Based on the passage, when the media now challenge the actions of a public official, the public assumes that:  
  • a)
    the official is always wrong.  
  • b)
    the media is always wrong.  
  • c)
    the media may be wrong.            
  • d)
    the official and the media may both be wrong.            
  • e)
    the public ignores this piece of news completely 
Correct answer is option 'C'. Can you explain this answer?

Kritika Bose answered
Explanation:

Understanding Public Perception:
- The passage highlights that the public's perception of the media has changed over time.
- The public now views the media with skepticism, similar to other large institutions in society.

Media Criticism of Public Officials:
- When the media now challenge the actions of a public official, the public assumes that the media may be wrong.
- This is because the passage mentions that many Americans have concluded that the media are no more credible than the government.
- Public opinion surveys reflect ambivalence about the press, indicating a lack of complete trust in the media.

Shift in Public Trust:
- The passage suggests that the media have lost some credibility due to scandals and public skepticism.
- As a result, when the media challenge public officials, the public does not automatically assume the official is wrong.
- Instead, there is a recognition that both the official and the media may be wrong in certain situations.
- This shift in public trust underscores the evolving relationship between the media, public officials, and the general public.

In the world today we make health and end in itself. We have forgotten that health is really means to enable a person to do his work and do it well. a lot of modern medicine and this includes many patients as well as many physicians pays very little attention to health but very much attention to those who imagine that they are ill. Our great concern with health is shown by the medical columns in newspapers. the health articles in popular magazines and the popularity of television programmes and all those books on medicine. We talk about health all the time. Yet for the most part the only result is more people with imaginary illness. The healthy man should not be wasting time talking about health: he should be using health for work. The work does the work that good health possible.
A healthy man should be concerned with
  • a)
    his work which good health makes possible
  • b)
    looking after his health
  • c)
    his health which makes work possible
  • d)
    talking about health
Correct answer is option 'A'. Can you explain this answer?

Nitya Tiwari answered
The correct answer is option 'A' - a healthy man should be concerned with his work which good health makes possible.

Explanation:

The passage explains that health is a means to enable a person to do his work and do it well. Therefore, the primary concern for a healthy man should be his work, which good health makes possible. The following points support this answer:

- Health is a means to an end: The passage highlights that health is not an end in itself but a means to enable a person to do his work well. Therefore, the focus should be on work rather than health.
- Modern medicine focuses on imaginary illnesses: The passage suggests that modern medicine and many patients and physicians pay little attention to health but focus on those who imagine that they are ill. This implies that the focus on health may not always be necessary or productive.
- Talking about health is not productive: The passage states that the great concern with health is shown by the media, but the only result is more people with imaginary illnesses. This suggests that talking about health may not lead to productive outcomes.
- Good health enables work: The passage emphasizes that good health enables a person to do his work well. Therefore, a healthy person should focus on his work rather than his health.

In summary, the passage suggests that a healthy man should be concerned with his work, which good health makes possible. While health is important, it is a means to an end rather than an end in itself.

In all battles two things are usually required of the Commander-in-Chief: to make a good plan for his army and to keep a strong reserve. Both of these are also obligatory for the painter. To make a plan, thorough reconnaissance of the country where the battle is to be fought is needed. Its fields, its mountains, its rivers, its bridges, its trees, its flowers, its atmosphere—all require and repay attentive observation from a special point of view.
I think this is one of the chief delights that have come to me through painting. No doubt many people who are lovers of art have acquired it to a high degree without actually practicing. But I expect that nothing will make one observe more quickly or more thoroughly than having to face the difficulty of representing the thing observed. And mind you, if you do observe accurately and with refinement, and if you do record what you have seen with tolerable correspondence, the result follows on the canvas with startling obedience.
But in order to make his plan, the General must not only reconnoitre the battle-ground; he must also study the achievements of the great Captains of the past. He must bring the observations he has collected in the field into comparison with the treatment of similar incidents by famous chiefs.
Considering this fact, the galleries of Europe take on a new—and to me at least — a severely practical interest. You see the difficulty that baffled you yesterday; and you see how easily it has been overcome by a great or even by a skilful painter. Not only is your observation of Nature sensibly improved and developed, but also your comprehension of the masterpieces of art.
But it is in the use and withholding of their reserves that the great commanders have generally excelled. After all, when once the last reserve has been thrown in, the commander‘s part is played. If that does not win the battle, he has nothing else to give. Everything must be left to luck and to the fighting troops. But these last reserves, in the absence of high direction, are apt to get into sad confusion, all mixed together in a nasty mess, without order or plan—and consequently without effect.  
Mere masses count no more. The largest brush, the brightest colours cannot even make an impression. The pictorial battlefield becomes a sea of mud mercifully veiled by the fog of war. Even though the General plunges in himself and emerges bespattered, as he sometimes does, he will not retrieve the day. In painting, the reserves consist in Proportion or Relation. And it is here that the art of the painter marches along the road which is traversed by all the greatest harmonies in thought. At one side of the palette there is white, at the other black; and neither is ever used 'neat.‘ Between these two rigid limits all the action must lie, all the power required must be generated. Black and white themselves placed in juxtaposition make no great impression; and yet they are the most that you can do in pure contrast.  
Directions: Read the above paragraph and answer the following
Q.Following the example of the master Manet, the young Matisse often inserted in his pictures areas of white such as tablecloths or crockery that allowed for striking contrasts with black objects such as a knife or a dark bottle. What is the relevance of this information to the passage?  
  • a)
    It supports the author‘s claim that the great artists are worthy of imitation.  
  • b)
    It supports the author‘s claim that neither black nor white is ever used ‗neat.‘
  • c)
    It weakens the author‘s claim that black and white themselves placed in juxtaposition make no great impression.  
  • d)
    It weakens the author‘s claim that great painters take Nature as their subject.  
  • e)
    This information has no relevance to the information in the passage 
Correct answer is option 'C'. Can you explain this answer?

Rithika Kaur answered
Mapping the Passage
¶1 compares painting a picture to fighting a battle, lists two similarities, planning and
backup, and discusses planning.
¶2 explains that practicing art is a great way to become a lover of art.
¶s3 and 4 draws in the analogy of the general and explain the need to study previous masters in war and art.
¶s5 and 6 explain the need to keep reserves in battle and painting.
A synthesis question testing your ability to evaluate the relevance of a new situation to the author‘s arguments. Zero in on elements of the new situation that sound relevant to the passage. Black and white are mentioned in the final paragraph. Recall that the author argues that black and white make weak impressions when contrasted. However, in the question stem situation, the impression is strong. We‘re looking for an answer that points this out, in other words, one that argues the new situation weakens the author‘s view (C) fits exactly.

The passage below is accompanied by four questions. Based on the passage, choose the best answer for each question.
Understanding romantic aesthetics is not a simple undertaking for reasons that are internal to the nature of the subject. Distinguished scholars, such as Arthur Lovejoy, Northrop Frye and Isaiah Berlin, have remarked on the notorious challenges facing any attempt to define romanticism. Lovejoy, for example, claimed that romanticism is “the scandal of literary history and criticism” . . . The main difficulty in studying the romantics, according to him, is the lack of any “single real entity, or type of entity” that the concept “romanticism” designates. Lovejoy concluded, “the word ‘romantic’ has come to mean so many things that, by itself, it means nothing” . . .
The more specific task of characterizing romantic aesthetics adds to these difficulties an air of paradox. Conventionally, “aesthetics” refers to a theory concerning beauty and art or the branch of philosophy that studies these topics. However, many of the romantics rejected the identification of aesthetics with a circumscribed domain of human life that is separated from the practical and theoretical domains of life. The most characteristic romantic commitment is to the idea that the character of art and beauty and of our engagement with them should shape all aspects of human life. Being fundamental to human existence, beauty and art should be a central ingredient not only in a philosophical or artistic life, but also in the lives of ordinary men and women. Another challenge for any attempt to characterize romantic aesthetics lies in the fact that most of the romantics were poets and artists whose views of art and beauty are, for the most part, to be found not in developed theoretical accounts, but in fragments, aphorisms and poems, which are often more elusive and suggestive than conclusive.
Nevertheless, in spite of these challenges the task of characterizing romantic aesthetics is neither impossible nor undesirable, as numerous thinkers responding to Lovejoy’s radical skepticism have noted. While warning against a reductive definition of romanticism, Berlin, for example, still heralded the need for a general characterization: “[Although] one does have a certain sympathy with Lovejoy’s despair…[he is] in this instance mistaken. There was a romantic movement…and it is important to discover what it is” . . .
Recent attempts to characterize romanticism and to stress its contemporary relevance follow this path. Instead of overlooking the undeniable differences between the variety of romanticisms of different nations that Lovejoy had stressed, such studies attempt to characterize romanticism, not in terms of a single definition, a specific time, or a specific place, but in terms of “particular philosophical questions and concerns” . . .
While the German, British and French romantics are all considered, the central protagonists in the following are the German romantics. Two reasons explain this focus: first, because it
has paved the way for the other romanticisms, German romanticism has a pride of place among the different national romanticisms . . . Second, the aesthetic outlook that was developed in Germany roughly between 1796 and 1801-02 — the period that corresponds to the heyday of what is known as “Early Romanticism” . . .— offers the most philosophical expression of romanticism since it is grounded primarily in the epistemological, metaphysical, ethical, and political concerns that the German romantics discerned in the aftermath of Kant’s philosophy.
Which one of the following statements is NOT supported by the passage?
  • a)
    Characterising romantic aesthetics is both possible and desirable, despite the challenges involved.
  • b)
    Recent studies on romanticism seek to refute the differences between national romanticisms.
  • c)
    Romantic aesthetics are primarily expressed through fragments, aphorisms, and poems.
  • d)
    Many romantics rejected the idea of aesthetics as a domain separate from other aspects of life.
Correct answer is option 'B'. Can you explain this answer?

Nitya Kumar answered
Explanation:

Recent studies on romanticism seek to refute the differences between national romanticisms
- This statement is NOT supported by the passage.
- According to the passage, recent studies do not seek to refute the differences between national romanticisms.
- Instead, these studies attempt to characterize romanticism in terms of particular philosophical questions and concerns.
- The passage emphasizes understanding romanticism in a broader sense without overlooking the differences between various national romanticisms.
Therefore, option B is not supported by the passage as it misrepresents the approach of recent studies on romanticism.

In all battles two things are usually required of the Commander-in-Chief: to make a good plan for his army and to keep a strong reserve. Both of these are also obligatory for the painter. To make a plan, thorough reconnaissance of the country where the battle is to be fought is needed. Its fields, its mountains, its rivers, its bridges, its trees, its flowers, its atmosphere—all require and repay attentive observation from a special point of view.
I think this is one of the chief delights that have come to me through painting. No doubt many people who are lovers of art have acquired it to a high degree without actually practicing. But I expect that nothing will make one observe more quickly or more thoroughly than having to face the difficulty of representing the thing observed. And mind you, if you do observe accurately and with refinement, and if you do record what you have seen with tolerable correspondence, the result follows on the canvas with startling obedience.
But in order to make his plan, the General must not only reconnoitre the battle-ground; he must also study the achievements of the great Captains of the past. He must bring the observations he has collected in the field into comparison with the treatment of similar incidents by famous chiefs.
Considering this fact, the galleries of Europe take on a new—and to me at least — a severely practical interest. You see the difficulty that baffled you yesterday; and you see how easily it has been overcome by a great or even by a skilful painter. Not only is your observation of Nature sensibly improved and developed, but also your comprehension of the masterpieces of art.
But it is in the use and withholding of their reserves that the great commanders have generally excelled. After all, when once the last reserve has been thrown in, the commander‘s part is played. If that does not win the battle, he has nothing else to give. Everything must be left to luck and to the fighting troops. But these last reserves, in the absence of high direction, are apt to get into sad confusion, all mixed together in a nasty mess, without order or plan—and consequently without effect.  
Mere masses count no more. The largest brush, the brightest colours cannot even make an impression. The pictorial battlefield becomes a sea of mud mercifully veiled by the fog of war. Even though the General plunges in himself and emerges bespattered, as he sometimes does, he will not retrieve the day. In painting, the reserves consist in Proportion or Relation. And it is here that the art of the painter marches along the road which is traversed by all the greatest harmonies in thought. At one side of the palette there is white, at the other black; and neither is ever used 'neat.‘ Between these two rigid limits all the action must lie, all the power required must be generated. Black and white themselves placed in juxtaposition make no great impression; and yet they are the most that you can do in pure contrast.  
Directions: Read the above paragraph and answer the following
Q.As the author creates the analogy between war and painting in the passage, the Commander-in-Chief is to the battleground as the:  
  • a)
    painter is to the subject being painted.  
  • b)
    painter is to the canvas of the painting.  
  • c)
    painter is to the paint colours.
  • d)
    painter is to the art gallery.  
  • e)
    painter is to the brush 
Correct answer is option 'A'. Can you explain this answer?

Mapping the Passage
¶1 compares painting a picture to fighting a battle, lists two similarities, planning and
backup, and discusses planning.
¶2 explains that practicing art is a great way to become a lover of art.
¶s3 and 4 draws in the analogy of the general and explain the need to study previous masters in war and art.
¶s5 and 6 explain the need to keep reserves in battle and painting.
The Commander-in-Chief is mentioned in the first paragraph, so begin your search there. The author says that the battleground must be inspected and studied. What is the equivalent in painting? The subject being painted. (A) fits the bill.

Henry Varnum Poor, editor of American Railroad Journal, drew the important elements of the image of the railroad together in 1851, ―Look at the results of this material progress...the vigor, life, and executive energy that followed in its train, rapidly succeeded by wealth, the refinement and intellectual culture of a high civilization. All this is typified, in a degree, by a locomotive. The combination in its construction of nice art and scientific application of power, its speed surpassing that of our proudest courser, and its immense strength, are all characteristic of our age and tendencies. To us, like the telegraph, it is essential, it constitutes a part of our nature, is a condition of our being what we are.
In the third decade of the nineteenth century, Americans began to define their character in light of the new railroads. They liked the idea that it took special people to foresee and capitalize on the promise of science. Railroad promoters, using the steam engine as a metaphor for what they thought Americans were and what they thought Americans were becoming, frequently discussed parallels between the locomotive and national character, pointing out that both possessed youth, power, speed, single-mindedness, and bright prospects.  
Poor was, of course, promoting acceptance of railroads and enticing his readers to open their pocketbooks. But his metaphors had their dark side. A locomotive was quite unlike anything Americans had ever seen. It was large, mysterious and dangerous; many thought that it was a monster waiting to devour the unwary. There was a suspicion that a country founded upon Jeffersonian agrarian principles had bought a ticket and boarded a train pulled by some iron monster into the dark recesses of an unknown future.
To ease such public apprehensions, promoters, poets, editors, and writers alike adopted the notion that locomotives were really only ―iron horses,‖ an early metaphor that lingered because it made steam technology ordinary and understandable. Iron horse metaphors assuaged fears about inherent defects in the national character, prompting images of a more secure future, and made an alien technology less frightening, and even comforting and congenial.  
Essayist Ralph Waldo Emerson saw the locomotive as an agent of domestic harmony. He observed that ―the locomotive and the steamboat, like enormous shuttles, shoot every day across the thousand various threads of national descent and employment and bind them fast in one web, "adding ―an hourly assimilation goes forward, and there is no danger that local peculiarities and hostilities should be preserved. To us Americans, it seems to have fallen as a political aid. We could not else have held the vast North America together, which we now engage to do"
Direction: Read the above Paragraph and answer the follownig Quetions
Q. The passage is primarily concerned with which of the following? 
  • a)
    criticise one interpretation of the early American railroads 
  • b)
    discuss the early years of the railroad and its connection to the American character of the time. 
  • c)
    suggest that railroads were the most important development in the history of America
  • d)
    describe the apprehension with which most of the Americans greeted the early railroads 
  • e)
    assert that Americans were tricked into believing that the railroads were beneficial for them 
Correct answer is option 'B'. Can you explain this answer?

Manasa Chavan answered
Mapping the Passage:
¶1 describes the opinions of one railroad promoter (Poor), who tied the railroad to the progressive nature of American character.
¶2 describes the American idea of the time that the railroad reflected elements of
American character.
¶s3 and 4 discuss the fears associated with the railroad and the metaphors presented to counter them.
¶5 describes the way that Americans were won over to the railroad by these metaphors (Emerson)
The passage broadly describes the early years of the railroad and its impact on the American character at that time. B fits in very nicely with this.
 

The passage below is accompanied by four questions. Based on the passage, choose the best answer for each question.
RESIDENTS of Lozère, a hilly department in southern France, recite complaints familiar to many rural corners of Europe. In remote hamlets and villages, with names such as Le Bacon and Le Bacon Vieux, mayors grumble about a lack of local schools, jobs, or phone and internet connections. Farmers of grazing animals add another concern: the return of wolves. Eradicated from France last century, the predators are gradually creeping back to more forests and hillsides. “The wolf must be taken in hand,” said an aspiring parliamentarian, Francis Palombi, when pressed by voters in an election campaign early this summer. Tourists enjoy visiting a wolf park in Lozère, but farmers fret over their livestock and their livelihoods. .
. .
As early as the ninth century, the royal office of the Luparii—wolf-catchers—was created in France to tackle the predators. Those official hunters (and others) completed their job in the 1930s, when the last wolf disappeared from the mainland. Active hunting and improved technology such as rifles in the 19th century, plus the use of poison such as strychnine later on, caused the population collapse. But in the early 1990s the animals reappeared. They crossed the Alps from Italy, upsetting sheep farmers on the French side of the border. Wolves have since spread to areas such as Lozère, delighting environmentalists, who see the predators’ presence as a sign of wider ecological health. Farmers, who say the wolves cause the deaths of thousands of sheep and other grazing animals, are less cheerful. They grumble that green activists and politically correct urban types have allowed the return of an old enemy.
Various factors explain the changes of the past few decades. Rural depopulation is part of the story. In Lozère, for example, farming and a once-flourishing mining industry supported a population of over 140,000 residents in the mid-19th century. Today the department has fewer than 80,000 people, many in its towns. As humans withdraw, forests are expanding. In France, between 1990 and 2015, forest cover increased by an average of 102,000 hectares each year, as more fields were given over to trees. Now, nearly one-third of mainland France is covered by woodland of some sort. The decline of hunting as a sport also means more forests fall quiet. In the mid-to-late 20th century over 2m hunters regularly spent winter weekends tramping in woodland, seeking boars, birds and other prey. Today the Fédération Nationale des Chasseurs, the national body, claims 1.1m people hold hunting licences, though the number of active hunters is probably lower. The mostly protected status of the wolf in Europe—hunting them is now forbidden, other than when occasional culls are sanctioned by the state—plus the efforts of NGOs to track and count the animals, also contribute to the recovery of wolf populations.
As the lupine population of Europe spreads westwards, with occasional reports of wolves seen closer to urban areas, expect to hear of more clashes between farmers and those who celebrate the predators’ return. Farmers’ losses are real, but are not the only economic story. Tourist venues, such as parks where wolves are kept and the animals’ spread is discussed, also generate income and jobs in rural areas.
  • a)
    lack of educational facilities.
  • b)
    poor rural communication infrastructure.
  • c)
    livestock losses.
  • d)
    decline in the number of hunting licences.
Correct answer is option 'D'. Can you explain this answer?

EduRev GMAT answered
Considering the first paragraph: “RESIDENTS of Lozère, a hilly department in southern France, recite complaints familiar to many rural corners of Europe. In remote hamlets and villages, with names such as Le Bacon and Le Bacon Vieux, mayors grumble about a lack of local schools, jobs, or phone and internet connections. Farmers of grazing animals add another concern: the return of wolves. Eradicated from France last century, the predators are gradually creeping back to more forests and hillsides. “The wolf must be taken in hand,” said an aspiring parliamentarian, Francis Palombi, when pressed by voters in an election campaign early this summer. Tourists enjoy visiting a wolf park in Lozère, but farmers fret over their livestock and their livelihoods. .”
 
Options A, B and C can be clearly inferred from the highlighted part.
The passage mentions that the number of people holding hunting licenses is still high but the number of people who still actively hunt is low. So Option D which states that there is decline in the number of hunting licences is incorrect.

The passage below is accompanied by four questions. Based on the passage, choose the best answer for each question.
RESIDENTS of Lozère, a hilly department in southern France, recite complaints familiar to many rural corners of Europe. In remote hamlets and villages, with names such as Le Bacon and Le Bacon Vieux, mayors grumble about a lack of local schools, jobs, or phone and internet connections. Farmers of grazing animals add another concern: the return of wolves. Eradicated from France last century, the predators are gradually creeping back to more forests and hillsides. “The wolf must be taken in hand,” said an aspiring parliamentarian, Francis Palombi, when pressed by voters in an election campaign early this summer. Tourists enjoy visiting a wolf park in Lozère, but farmers fret over their livestock and their livelihoods. .
. .
As early as the ninth century, the royal office of the Luparii—wolf-catchers—was created in France to tackle the predators. Those official hunters (and others) completed their job in the 1930s, when the last wolf disappeared from the mainland. Active hunting and improved technology such as rifles in the 19th century, plus the use of poison such as strychnine later on, caused the population collapse. But in the early 1990s the animals reappeared. They crossed the Alps from Italy, upsetting sheep farmers on the French side of the border. Wolves have since spread to areas such as Lozère, delighting environmentalists, who see the predators’ presence as a sign of wider ecological health. Farmers, who say the wolves cause the deaths of thousands of sheep and other grazing animals, are less cheerful. They grumble that green activists and politically correct urban types have allowed the return of an old enemy.
Various factors explain the changes of the past few decades. Rural depopulation is part of the story. In Lozère, for example, farming and a once-flourishing mining industry supported a population of over 140,000 residents in the mid-19th century. Today the department has fewer than 80,000 people, many in its towns. As humans withdraw, forests are expanding. In France, between 1990 and 2015, forest cover increased by an average of 102,000 hectares each year, as more fields were given over to trees. Now, nearly one-third of mainland France is covered by woodland of some sort. The decline of hunting as a sport also means more forests fall quiet. In the mid-to-late 20th century over 2m hunters regularly spent winter weekends tramping in woodland, seeking boars, birds and other prey. Today the Fédération Nationale des Chasseurs, the national body, claims 1.1m people hold hunting licences, though the number of active hunters is probably lower. The mostly protected status of the wolf in Europe—hunting them is now forbidden, other than when occasional culls are sanctioned by the state—plus the efforts of NGOs to track and count the animals, also contribute to the recovery of wolf populations.
As the lupine population of Europe spreads westwards, with occasional reports of wolves seen closer to urban areas, expect to hear of more clashes between farmers and those who celebrate the predators’ return. Farmers’ losses are real, but are not the only economic story. Tourist venues, such as parks where wolves are kept and the animals’ spread is discussed, also generate income and jobs in rural areas.
Which one of the following has NOT contributed to the growing wolf population in Lozère?
  • a)
    A decline in the rural population of Lozère.
  • b)
    An increase in woodlands and forest cover in Lozère.
  • c)
    The shutting down of the royal office of the Luparii.
  • d)
    The granting of a protected status to wolves in Europe.
Correct answer is option 'C'. Can you explain this answer?

EduRev GMAT answered
The passage mentions that as early as the ninth century, the royal office of the Luparii, or wolf-catchers, was created in France to tackle the predators. However, this office became redundant as it had finished it’s job (kill the last wolf). So the resurgence of the wolfs can’t be attributed to it shutting down. The other options on the other hand, can be clearly inferred.
Option A: “Various factors explain the changes of the past few decades. Rural depopulation is part of the story. In Lozère, for example, farming and a once-flourishing mining industry supported a population of over 140,000 residents in the mid-19th century. Today the department has fewer than 80,000 people, many in its towns. “
Option B: “As humans withdraw, forests are expanding. In France, between 1990 and 2015, forest cover increased by an average of 102,000 hectares each year, as more fields were given over to trees. Now, nearly one-third of mainland France is covered by woodland of some sort. “
Option D: “The mostly protected status of the wolf in Europe—hunting them is now forbidden, other than when occasional culls are sanctioned by the state—plus the efforts of NGOs to track and count the animals, also contribute to the recovery of wolf populations.”

Once surrounded and protected by vast wilderness, many of the national parks are adversely affected by activities outside their boundaries. The National Park Organic Act established the national park system and empowered the Secretary of the Interior to manage activities within the parks. Conditions outside park boundaries are not subject to regulation by the Park Service unless they involve the direct use of park resources.  
Several approaches to protecting the national parks from external degradation have been proposed, such as one focusing on enacting federal legislation granting the National Park Service broader powers over lands adjacent to the national parks. Legislation addressing external threats to the national parks twice passed the House of Representatives but died without action in the Senate. Also brought to the table as a possible remedy is giving the states bordering the parks a significant and meaningful role in developing federal park management policy.
Because the livelihood of many citizens is linked to the management of national parks, local politicians often encourage state involvement in federal planning. But, state legislatures have not always addressed the fundamental policy issues of whether states should protect park wildlife.  
Timber harvesting, ranching and energy exploration compete with wildlife within the local ecosystem. Priorities among different land uses are not generally established by current legislation. Additionally, often no mechanism exists to coordinate planning by the state environmental regulatory agencies. These factors limit the impact of legislation aimed at protecting park wildlife and the larger park ecosystem.  
Even if these deficiencies can be overcome, state participation must be consistent with existing federal legislation. States lack jurisdiction within national parks themselves, and therefore state solutions cannot reach activities inside the parks, thus limiting state action to the land adjacent to the national parks. Under the supremacy clause, federal laws and regulations supersede state action if state law conflicts with federal legislation, if Congress precludes local regulation, or if federal regulation is so pervasive that no room remains for state control. Assuming that federal regulations leave open the possibility of state control, state participation in policy making must be harmonized with existing federal legislation.  
The residents of states bordering national parks are affected by park management policies. They in turn affect the success of those policies. This interrelationship must be considered in responding to the external threats problem. Local participation is necessary in deciding how to protect park wildlife. Local interests should not, however, dictate national policy, nor should they be used as a pretext to ignore the threats to park regions. 
Direction: Read the above Paragraph and answer the follownig Quetions
Q. According to the passage, which of the following developments is most likely if environmental cooperation between the federal government and state governments does not improve? 
  • a)
    A further decline in the land area of national parks  
  • b)
    A further increase in federal ownership of land adjacent to national parks  
  • c)
    A further growth in the powers of the National Park Service  
  • d)
    A further loss of species in national parks
  • e)
    A further increase in timber harvesting activities 
Correct answer is option 'D'. Can you explain this answer?

Prisha Shah answered
Mapping the Passage:
¶1 describes a problem facing national parks: negative effects from the land
surrounding them.
¶2 describes one approach to dealing with the problem: federal legislation, which
failed.
¶3 and 4 describe a second approach: giving power to states to cooperate with
adjacent national parks, and describe the problems with it.
¶5 argues that state participation must be tied to federal regulations.
¶6 argues that any solution requires a national response with elements of local participation.
The ―according to the passage...‖ start to the question tips you off to look for a dnesetail within the passage. Where is the scenario in the question mentioned? Go to the last paragraph, which discusses a combination of national and local responses. It argues that this cooperation is necessary in order to ―protect park wildlife.If this cooperation doesn‘t occur then, wildlife would presumably be
harmed. (D) rewards the careful reading.

In all battles two things are usually required of the Commander-in-Chief: to make a good plan for his army and to keep a strong reserve. Both of these are also obligatory for the painter. To make a plan, thorough reconnaissance of the country where the battle is to be fought is needed. Its fields, its mountains, its rivers, its bridges, its trees, its flowers, its atmosphere—all require and repay attentive observation from a special point of view.
I think this is one of the chief delights that have come to me through painting. No doubt many people who are lovers of art have acquired it to a high degree without actually practicing. But I expect that nothing will make one observe more quickly or more thoroughly than having to face the difficulty of representing the thing observed. And mind you, if you do observe accurately and with refinement, and if you do record what you have seen with tolerable correspondence, the result follows on the canvas with startling obedience.
But in order to make his plan, the General must not only reconnoitre the battle-ground; he must also study the achievements of the great Captains of the past. He must bring the observations he has collected in the field into comparison with the treatment of similar incidents by famous chiefs.
Considering this fact, the galleries of Europe take on a new—and to me at least — a severely practical interest. You see the difficulty that baffled you yesterday; and you see how easily it has been overcome by a great or even by a skilful painter. Not only is your observation of Nature sensibly improved and developed, but also your comprehension of the masterpieces of art.
But it is in the use and withholding of their reserves that the great commanders have generally excelled. After all, when once the last reserve has been thrown in, the commander‘s part is played. If that does not win the battle, he has nothing else to give. Everything must be left to luck and to the fighting troops. But these last reserves, in the absence of high direction, are apt to get into sad confusion, all mixed together in a nasty mess, without order or plan—and consequently without effect.  
Mere masses count no more. The largest brush, the brightest colours cannot even make an impression. The pictorial battlefield becomes a sea of mud mercifully veiled by the fog of war. Even though the General plunges in himself and emerges bespattered, as he sometimes does, he will not retrieve the day. In painting, the reserves consist in Proportion or Relation. And it is here that the art of the painter marches along the road which is traversed by all the greatest harmonies in thought. At one side of the palette there is white, at the other black; and neither is ever used 'neat.‘ Between these two rigid limits all the action must lie, all the power required must be generated. Black and white themselves placed in juxtaposition make no great impression; and yet they are the most that you can do in pure contrast.  
Directions: Read the above paragraph and answer the following
Q.The author‘s statement ―"But [the fighting troops], in the absence of high direction, are apt to get into sad confusion, all mixed together in a nasty mess, without order or plan—and consequently without effect" assumes that
  • a)
    chaotic painting cannot have an unintended artistic effect.  
  • b)
    an artist naturally resists direction from another individual.  
  • c)
    a painting cannot help but reflect the mental state of its painter.  
  • d)
    it is impossible for painters to collaborate on a work without confusion.
  • e)
    troops always need someone to guide them 
Correct answer is option 'A'. Can you explain this answer?

Explanation:

Understanding the Authors Statement:
The author's statement highlights the importance of direction and leadership in both military strategy and painting. The comparison drawn between a commander-in-chief leading troops in battle and a painter creating a work of art emphasizes the need for a clear plan and guidance to achieve success.

Interpreting the Assumption:
The assumption made in the statement is that chaotic painting, without a clear direction or plan from the artist, will not result in a meaningful or impactful artistic effect. This assumption suggests that without proper guidance and order, the creative process may lead to confusion and a lack of coherence in the final artwork.

Relation to the Military Analogy:
In the military context, the absence of high direction or leadership can result in troops becoming disorganized and ineffective in battle. Similarly, in painting, without a clear vision or plan from the artist, the artwork may lack structure and fail to convey the intended message or emotion.

Significance of Guided Creativity:
The author emphasizes the importance of having a plan and utilizing reserves effectively in both warfare and art. By following a strategic approach and avoiding chaos or disarray, artists can create more impactful and meaningful works of art.

Conclusion:
In essence, the assumption underlying the author's statement is that without proper direction and guidance, whether in warfare or painting, the outcome may be chaotic and lack the intended impact or effectiveness. This highlights the importance of leadership, planning, and organization in achieving success in any creative endeavor.

As formal organizations, business corporations are distinguished by their particular goals, which include maximization of profits, growth, and survival. Providing goods and services is a means to this end. If, for example, a number of individuals (outsiders or even insiders) believe that a company‘s aggressive marketing of infant formula in third world countries is morally wrong, the company is unlikely to be moved by arguments based on ethos alone as long as what it is doing remains profitable. But if those opposed to the company‘s practice organize a highly effective boycott of the company‘s products, their moral views will soon enter into the company‘s deliberations indirectly as limiting operating conditions. They can, at this point, no more be ignored than a prohibitive increase in the costs of certain raw materials.  
Although the concepts and categories of ethics may be applied to the conduct of corporations, there are important differences between the values and principles underlying corporate behaviour and those underlying the actions of most individuals. If corporations are by their nature end- or goal-directed how can they acknowledge acts as wrong in and of themselves? Is it possible to hold one criminally responsible for acts that if performed by a human person would result in criminal liability?  
The first case of this type to achieve widespread public attention was the attempt to prosecute the Ford Motor Company for manslaughter as the result of alleged negligent or reckless decision making concerning the safety engineering of the Pinto vehicle. Although the defendant corporation and its officers were found innocent after trial, the case can serve as an exemplar for our purposes.  
In essence, the prosecution in this case attempted to show that the corporation had produced and distributed a vehicle that was known to be defective at the time of production and sale, and that even after a great deal of additional information accumulated regarding the nature of the problems, the corporation took no action to correct them. The obvious non-corporate analogy would be the prosecution of a person who was driving a car with brakes known to be faulty, who does not have them repaired because it would cost too much, and who kills someone when the brakes eventually fail and the car does not stop in time. Such cases involving individuals are prosecuted and won regularly.
If corporations have no concept of right or wrong because they are exclusively goal-directed, can they be convicted in cases of this type, and what purpose would be served by such a conviction? Perhaps we can make a utilitarian argument for convicting corporations of such crimes. The argument would be that of deterrence; conviction and punishment would deter other corporations from taking similar actions under similar circumstances. However, there appears to be considerable evidence that deterrence does not work on corporations, even if, arguably, it works on individuals. The possibility of being discovered and the potential magnitude of the fine merely become more data to be included in the analysis of limiting conditions.  
Directions: Read the above paragraph and answer the following:
Q.A claim that things have ethical value to corporations only insofar as they are instrumental in furthering the ultimate goals of the corporation is:  
  • a)
    necessarily true, given the information presented in the passage.  
  • b)
    perhaps true, and supported by the information presented in the passage.  
  • c)
    perhaps true, but not supported by any information in the passage.  
  • d)
    necessarily false, given the information presented in the passage.  
  • e)
    a figment of the author‘s imagination
Correct answer is option 'B'. Can you explain this answer?

Arpita Basu answered
Explanation:

Claim about ethical value to corporations:
- The claim that things have ethical value to corporations only insofar as they are instrumental in furthering the ultimate goals of the corporation is perhaps true, and it is supported by the information presented in the passage.
- The passage discusses how corporations are distinguished by their goals of profit maximization, growth, and survival, and how providing goods and services is a means to achieve these goals.
- It also mentions that corporations may not be moved by ethical arguments alone if their actions remain profitable, indicating that ethical considerations are secondary to their goals.

Support from the passage:
- The passage states that corporations are goal-directed entities, focused on achieving their objectives, which may not inherently include ethical values.
- It discusses how corporations may only consider moral views when they become limiting factors, similar to cost increases in raw materials.
- The example of the Ford Motor Company case further illustrates how corporations prioritize goals over ethical considerations.
Therefore, based on the information presented in the passage, the claim that ethical value to corporations is instrumental in furthering their goals is perhaps true and supported by the context provided.

Suspicious as they are of American intentions, and bolstered by court rulings that seem to give them license to seek out and publish any and all government secrets, the media‘s distrust of our government, combined with their limited understanding of the world at large, damages our ability to design and conduct good policy in ways that the media rarely imagine.  
The leak through which sensitive information flows from the government to the press is detrimental to policy in so far as it almost completely precludes the possibility of serious discussion. The fear that anything they say, even in what is construed as a private forum, may appear in print, makes many people, whether our own government officials or the leaders of foreign countries, unwilling to speak their minds. 
Must we be content with the restriction of our leaders‘ policy discussions to a handful of people who trust each other, thus limiting the richness and variety of ideas that could be brought forward through a larger group because of the nearly endemic nature of this problem? It is vitally important for the leaders of the United States to know the real state of affairs internationally, and this can occur only if foreign leaders feel free to speak their minds to our diplomats.  
Until recently, it looked as if the media had convinced the public that journalists were more reliable than the government; however, this may be changing. With the passage of time, the media have lost lustre. They—having grown large and powerful—provoke the same public skepticism that other large institutions in the society do. A series of media scandals has contributed to this. Many Americans have concluded that the media are no more credible than the government, and public opinion surveys reflect much ambivalence about the press.
While leaks are generally defended by media officials on the grounds of the public‘s ―right to know,‖ in reality they are part of the Washington political power game, as well as part of the policy process. The "leaker" may be currying favour with the media, or may be planting information to influence policy. In the first case, he is helping himself by enhancing the prestige of a journalist; in the second, he is using the media as a stage for his preferred policies. In either instance, it closes the circle: the leak begins with a political motive, is advanced by a politicized media, and continues because of politics. Although some of the journalists think they are doing the work, they are more often than not instruments of the process, not prime movers. The media must be held accountable for their activities, just like every other significant institution in our society, and the media must be forced to earn the public‘s trust. 
Direction: Read the above Paragraph and answer the follownig Quetions
Q. Implicit in the author‘s argument that leaks result in far more limited and unreliable policy discussions with foreign leaders is the idea that:  
  • a)
    leaks should be considered breaches of trust and therefore immoral. 
  • b)
    leaks have occurred throughout the history of politics. 
  • c)
    foreign and U.S. leaders discussed policy without inhibition before the rise of the mass media.  
  • d)
    leaders fear the public would react negatively if it knew the real state of affairs.  
  • e)
    it is best to keep the media in the dark 
Correct answer is option 'D'. Can you explain this answer?

Harshad Iyer answered
Mapping the Passage
¶1 argues that the media‘s suspicion of government and lack of knowledge about the world harm government policy.
¶s2 and 3 introduce the concept of the ―leak‖ and explain why it‘s bad for foreign policy.
¶4 states that the media was trusted by the public until recently, but are now met with skepticism.
¶5 argues that leaks are usually part of a power grab and that the media is a pawn in the game.
This question requires students to find the assumption in the lines mentioned. Review the author‘s argument in ¶2 that leaks harm discussions with foreign leaders. What is the author assuming in this argument? The author argues that foreign leaders don‘t want their private thoughts to be made public; he must also therefore assume that leaders have some sort of reason for not wanting their views to be made public. (D) provides a possible reason. If unclear, use the denial test: if leaders didn‘t have this fear, what would be their motivation for hiding their personal views?

In public Greek life, a man had to make his way at every step through the immediate persuasion of the spoken word. Whether it be addressing an assembly, a law-court or a more restricted body, his oratory would be a public affair rather than under the purview of a quiet committee, without the support of circulated commentary, and with no backcloth of daily reportage to make his own or others‘ views familiar to his hearers. The oratory's immediate effect was all-important; it would be naive to expect that mere reasonableness or an inherently good case would equate to a satisfactory appeal. Therefore, it was early realized that persuasion was an art, up to a point teachable, and a variety of specific pedagogy was well established in the second half of the fifth century. When the sophists claimed to teach their pupils how to succeed in public life, rhetoric was a large part of what they meant, though, to do them justice, it was not the whole.
Skill naturally bred mistrust. If a man of good will had need of expression advanced of mere twaddle, to learn how to expound his contention effectively, the truculent or pugnacious could be taught to dress their case in well-seeming guise. It was a standing charge against the sophists that they ‗made the worse appear the better cause,‘ and it was this immoral lesson which the hero of Aristophanes‘ Clouds went to learn from, of all people, Socrates. Again, the charge is often made in court that the opponent is an adroit orator and the jury must be circumspect so as not to let him delude them. From the frequency with which this crops up, it is patent that the accusation of cleverness might damage a man. In Greece, juries, of course, were familiar with the style, and would recognize the more evident artifices, but it was worth a litigant‘s while to get his speech written for him by an expert. Persuasive oratory was certainly one of the pressures that would be effective in an Athenian law-court.
A more insidious danger was the inevitable desire to display this art as an art. It is not easy to define the point at which a legitimate concern with style shades off into preoccupation with manner at the expense of matter, but it is easy to perceive that many Greek writers of the fourth and later centuries passed that danger point. The most influential was Isocrates, who polished for long years his pamphlets, written in the form of speeches, and taught to many pupils the smooth and easy periods he had perfected. Isocrates took to the written word in compensation for his inadequacy in live oratory; the tough and nervous tones of a Demosthenes were far removed from his, though they, too, were based on study and practice. The exaltation of virtuosity did palpable harm. The balance was always delicate, between style as a vehicle and style as an end in itself.
 We must not try to pinpoint a specific moment when it, once and for all, tipped over; but certainly, as time went on, virtuosity weighed heavier. While Greek freedom lasted, and it mattered what course of action a Greek city decided to take, rhetoric was a necessary preparation for public life, whatever its side effects. It had been a source of strength for Greek civilization that its problems, of all kinds, were thrashed out very much in public. The shallowness which the study of rhetoric might (not must) encourage was the corresponding weakness.  
Directions: Read the above paragraph and answer the following
Q.If the author of the passage travelled to a political convention and saw various candidates speak he would most likely have the highest regard for an orator who:  
  • a)
    roused his hearers to immediate and decisive action.  
  • b)
    understood that rhetoric serves an aesthetic as well as a practical purpose.  
  • c)
    relied on facts and reason rather than on rhetorical devices in making his case.  
  • d)
    passed on the techniques he had perfected to many students.  
  • e)
    made use of flowery and inflated words 
Correct answer is option 'C'. Can you explain this answer?

Mapping the Passage:
¶1 introduces the importance of rhetoric in Greek life and the fact that it was taught.
¶2 explains why rhetorical skill was sometimes mistrusted, but still sought after.
¶3 states that as rhetoric (in writing and speech) became more of an art, its original purpose was corrupted.
¶4 states that the Greek system required rhetorical skills and therefore inherited rhetoric's drawbacks as well.
What is the author‘s main argument about oratory? It was necessary for the Greeks, but became a "weakness" when they focused too much on making it artistic (¶s 3 and 4). Therefore the author would admire an orator who didn't sacrifice the facts and reason to too much rhetoric. (C) keeps the good parts of rhetoric while leaving out the artistic flourishes the author dislikes.

The ―paradox of tolerance‖ admonishes us that tolerance of the intolerant leads to intolerance. The Universal Declaration of Human Rights and the constitutions and laws of Western European democracies that adhere to the principle of freedom of speech all heed the warning of this conundrum and do not afford legal protection to extremist speech.  
While in Western European democracies, the speech of nondemocratic extremists has been successfully outlawed, in the United States the first amendment right to freedom of speech has been interpreted to encompass radical oration. The traditional justifications of this American stance originate in the belief that speech is entitled to greater tolerance than other kinds of activity. They are based on the belief that speech itself is valuable, and thus ascribe positive value to a very broad range of speech.
 According to the classical model, freedom of speech serves an indispensable function in the process of democratic self-government. From this perspective, the free speech principle need only protect political speech, comprised of all the facts, theories, and opinions relating to any issue on which the citizens must vote. Proponents of this view insist that even extremist views cannot be concealed from voting citizens, if these views bear on any public issue before them.
Protection of free speech serves the collective self-interests of a selfgoverning society made up of all rational, equal, and fully participating citizens who take their civic duties seriously. The fortress model is built on a foundation of pessimism, individualism, relativism, and self-doubt. At its deepest level, the fortress model values freedom of speech as a necessary precondition to the discovery and preservation of truth, but even at this level the function of speech remains primarily negative.  
From this perspective, the government and a majority of the people pose a great danger of intolerance. In spite of the high probability that their beliefs will eventually prove to be false, it is argued, people nonetheless tend to feel certain about them and, consequently, feel justified in requiring others to conform. Thus, the fortress model‘s prescription for combating the tendency to censor nonconforming views is to overprotect speech by providing a broad ―buffer zone‖ that encompasses extremist speech because its protection substantially diminishes the probability that inherently valuable speech will be suppressed.  
Directions: Read the above paragraph and answer the following
Q. Which of the following scenarios, if true, would most weaken the argument contained in the paradox of tolerance which ―admonishes us that tolerance of the intolerant leads to intolerance?‖ 
  • a)
    Islandia‘s government has decided to outlaw extremist political groups in order to protect its democratic political system.  
  • b)
    Islandia has a non-democratic government, despite its suppression of extremist political groups.  
  • c)
    Islandia‘s government became democratic only after extremist political groups were outlawed.  
  • d)
    Islandia has had a stable democratic government for decades, even though it has never outlawed extremist political groups  
  • e)
    Islandia has tried and failed repeatedly to have a stable government 
Correct answer is option 'D'. Can you explain this answer?

Ameya Sen answered
Mapping the Passage
¶1 introduces the ―"paradox of intolerance."
¶2 gives examples of countries that have taken the paradox into consideration by banning extreme speech, and argues that the United States tolerates even extremist speech.
¶3 introduces a justification for the defense of extremist speech: the classical model.
¶s4 and 5 introduce another justification: the fortress model.
Take a moment to review the paradox and its implications before attacking the choices. The paradox essentially states that free speech should be limited at its extremes when the extremes could contribute to eliminating free speech. Looking for an answer choice that weakens the implications of the paradox turns up (D): The paradox states that extreme speech weakens stable government, a point weakened by a scenario in which extreme speech and stable government coexist.

The passage below is accompanied by four questions. Based on the passage, choose the best answer for each question.
RESIDENTS of Lozère, a hilly department in southern France, recite complaints familiar to many rural corners of Europe. In remote hamlets and villages, with names such as Le Bacon and Le Bacon Vieux, mayors grumble about a lack of local schools, jobs, or phone and internet connections. Farmers of grazing animals add another concern: the return of wolves. Eradicated from France last century, the predators are gradually creeping back to more forests and hillsides. “The wolf must be taken in hand,” said an aspiring parliamentarian, Francis Palombi, when pressed by voters in an election campaign early this summer. Tourists enjoy visiting a wolf park in Lozère, but farmers fret over their livestock and their livelihoods. .
. .
As early as the ninth century, the royal office of the Luparii—wolf-catchers—was created in France to tackle the predators. Those official hunters (and others) completed their job in the 1930s, when the last wolf disappeared from the mainland. Active hunting and improved technology such as rifles in the 19th century, plus the use of poison such as strychnine later on, caused the population collapse. But in the early 1990s the animals reappeared. They crossed the Alps from Italy, upsetting sheep farmers on the French side of the border. Wolves have since spread to areas such as Lozère, delighting environmentalists, who see the predators’ presence as a sign of wider ecological health. Farmers, who say the wolves cause the deaths of thousands of sheep and other grazing animals, are less cheerful. They grumble that green activists and politically correct urban types have allowed the return of an old enemy.
Various factors explain the changes of the past few decades. Rural depopulation is part of the story. In Lozère, for example, farming and a once-flourishing mining industry supported a population of over 140,000 residents in the mid-19th century. Today the department has fewer than 80,000 people, many in its towns. As humans withdraw, forests are expanding. In France, between 1990 and 2015, forest cover increased by an average of 102,000 hectares each year, as more fields were given over to trees. Now, nearly one-third of mainland France is covered by woodland of some sort. The decline of hunting as a sport also means more forests fall quiet. In the mid-to-late 20th century over 2m hunters regularly spent winter weekends tramping in woodland, seeking boars, birds and other prey. Today the Fédération Nationale des Chasseurs, the national body, claims 1.1m people hold hunting licences, though the number of active hunters is probably lower. The mostly protected status of the wolf in Europe—hunting them is now forbidden, other than when occasional culls are sanctioned by the state—plus the efforts of NGOs to track and count the animals, also contribute to the recovery of wolf populations.
As the lupine population of Europe spreads westwards, with occasional reports of wolves seen closer to urban areas, expect to hear of more clashes between farmers and those who celebrate the predators’ return. Farmers’ losses are real, but are not the only economic story. Tourist venues, such as parks where wolves are kept and the animals’ spread is discussed, also generate income and jobs in rural areas.
The inhabitants of Lozère have to grapple with all of the following problems, EXCEPT:
  • a)
    lack of educational facilities.
  • b)
    poor rural communication infrastructure.
  • c)
    livestock losses.
  • d)
    decline in the number of hunting licences.
Correct answer is option 'D'. Can you explain this answer?

EduRev GMAT answered
  • Considering the first paragraph: “RESIDENTS of Lozère, a hilly department in southern France, recite complaints familiar to many rural corners of Europe. In remote hamlets and villages, with names such as Le Bacon and Le Bacon Vieux, mayors grumble about a lack of local schools, jobs, or phone and internet connections. Farmers of grazing animals add another concern: the return of wolves. Eradicated from France last century, the predators are gradually creeping back to more forests and hillsides. “The wolf must be taken in hand,” said an aspiring parliamentarian, Francis Palombi, when pressed by voters in an election campaign early this summer. Tourists enjoy visiting a wolf park in Lozère, but farmers fret over their livestock and their livelihoods. .”
  • Options A, B and C can be clearly inferred from the highlighted part.
  • The passage mentions that the number of people holding hunting licenses is still high but the number of people who still actively hunt is low. So Option D which states that there is decline in the number of hunting licences is incorrect.

Direction: Read the following paragraph carefully and answer the question given below:
Suspicious as they are of American intentions, and bolstered by court rulings that seem to give them license to seek out and publish any and all government secrets, the media‘s distrust of our government, combined with their limited understanding of the world at large, damages our ability to design and conduct good policy in ways that the media rarely imagine.
The leak through which sensitive information flows from the government to the press is detrimental to policy in so far as it almost completely precludes the possibility of serious discussion. The fear that anything they say, even in what is construed as a private forum, may appear in print, makes many people, whether our own government officials or the leaders of foreign countries, unwilling to speak their minds.
Must we be content with the restriction of our leaders‘ policy discussions to a handful of people who trust each other, thus limiting the richness and variety of ideas that could be brought forward through a larger group because of the nearly endemic nature of this problem? It is vitally important for the leaders of the United States to know the real state of affairs internationally, and this can occur only if foreign leaders feel free to speak their minds to our diplomats.
Until recently, it looked as if the media had convinced the public that journalists were more reliable than the government; however, this may be changing. With the passage of time, the media have lost lustre. They—having grown large and powerful—provoke the same public skepticism that other large institutions in the society do. A series of media scandals has contributed to this. Many Americans have concluded that the media are no more credible than the government, and public opinion surveys reflect much ambivalence about the press.
While leaks are generally defended by media officials on the grounds of the public‘s ―right to know,in reality they are part of the Washington political power game, as well as part of the policy process. The "leaker" may be currying favour with the media, or may be planting information to influence policy. In the first case, he is helping himself by enhancing the prestige of a journalist; in the second, he is using the media as a stage for his preferred policies. In either instance, it closes the circle: the leak begins with a political motive, is advanced by a politicized media, and continues because of politics. Although some of the journalists think they are doing the work, they are more often than not instruments of the process, not prime movers. The media must be held accountable for their activities, just like every other significant institution in our society, and the media must be forced to earn the public‘s trust.
Based on the information in the passage, with which of the following statements would the author most likely agree? 
  • a)
    Feeding the public misinformation is warranted in certain situations.  
  • b)
    The public has a right to know the real state of foreign affairs.  
  • c)
    The fewer the number of people involved in policy discussions, the better.  
  • d)
    Leaders give up their right to privacy when they are elected.  
  • e)
    The media is not accountable to the public. 
Correct answer is option 'A'. Can you explain this answer?

Gowri Bose answered
Option: 1
Explanation:
Review the author's main arguments before looking for an answer choice that he's agree with. (A) recalls the author's point in paragraph 2: "Leaders often say one thing in public and something quite different in public conversation... " The author explains why this occurs—fear of media leaks—and clearly opposes such leaks. Therefore, the author must agree with (A)'s contention that misinformation is sometimes warranted.
Wrong Answers:
(B): Opposite. This is the opposite of (A); for the same reasons that (A) is a valid inference, (B) isn't.
(C): Opposite. The author argues in paragraph 3 that policy benefits from a "richness and variety of ideas. "
(D): Opposite. The author's point in decrying leaks is that privacy is a necessary component

Henry Varnum Poor, editor of American Railroad Journal, drew the important elements of the image of the railroad together in 1851, ―Look at the results of this material progress...the vigor, life, and executive energy that followed in its train, rapidly succeeded by wealth, the refinement and intellectual culture of a high civilization. All this is typified, in a degree, by a locomotive. The combination in its construction of nice art and scientific application of power, its speed surpassing that of our proudest courser, and its immense strength, are all characteristic of our age and tendencies. To us, like the telegraph, it is essential, it constitutes a part of our nature, is a condition of our being what we are.
 In the third decade of the nineteenth century, Americans began to define their character in light of the new railroads. They liked the idea that it took special people to foresee and capitalize on the promise of science. Railroad promoters, using the steam engine as a metaphor for what they thought Americans were and what they thought Americans were becoming, frequently discussed parallels between the locomotive and national character, pointing out that both possessed youth, power, speed, single-mindedness, and bright prospects.
 Poor was, of course, promoting acceptance of railroads and enticing his readers to open their pocketbooks. But his metaphors had their dark side. A locomotive was quite unlike anything Americans had ever seen. It was large, mysterious and dangerous; many thought that it was a monster waiting to devour the unwary. There was a suspicion that a country founded upon Jeffersonian agrarian principles had bought a ticket and boarded a train pulled by some iron monster into the dark recesses of an unknown future.  T
To ease such public apprehensions, promoters, poets, editors, and writers alike adopted the notion that locomotives were really only ―iron horses,‖ an early metaphor that lingered because it made steam technology ordinary and understandable. Iron horse metaphors assuaged fears about inherent defects in the national character, prompting images of a more secure future, and made an alien technology less frightening, and even comforting and congenial.  
Essayist Ralph Waldo Emerson saw the locomotive as an agent of domestic harmony. He observed that ―the locomotive and the steamboat, like enormous shuttles, shoot every day across the thousand various threads of national descent and employment and bind them fast in one web,"adding ―an hourly assimilation goes forward, and there is no danger that local peculiarities and hostilities should be preserved. To us Americans, it seems to have fallen as a political aid. We could not else have held the vast North America together, which we now engage to do."
Direction: Read the above Paragraph and answer the follownig Quetions
Q. Suppose that an early nineteenth-century American inventor had developed a device that made it easier to construct multi-story building. How would early nineteenth-century Americans be expected to react to this invention?  
  • a)
    They would not support society‘s use of such a device.
  • b)
    They would generally support society‘s use of such a device.        
  • c)
    They would have no opinion about society‘s use of such a device.
  • d)
    They themselves would not use such a device.  
  • e)
    They would initially view such a device with skepticism 
Correct answer is option 'B'. Can you explain this answer?

Mrinalini Basu answered
Mapping the Passage:
¶1 describes the opinions of one railroad promoter (Poor), who tied the railroad to the progressive nature of American character.
¶2 describes the American idea of the time that the railroad reflected elements of American character.
¶s3 and 4 discuss the fears associated with the railroad and the metaphors presented to counter them.
¶5 describes the way that Americans were won over to the railroad by these metaphors (Emerson).
The new situation involves scientific progress much like the railroad; what does the author say about Americans‘ ideas about this? Go back to ¶2: the author argues that Americans had a special fondness for science and progress. Therefore, they‘d endorse something that furthered these goals. (B) fits.

By regarding the expanding universe as a motion picture, you can easily imagine ―running the film backward.‖ If you do so, you find the universe getting smaller and smaller, and eventually you come to the moment when its whole mass is crammed into an infinitely dense point. Before that time it didn‘t exist, or at least it didn‘t exist in its present form. Though there is some controversy about its exact age, most cosmologists would be inclined to agree that the universe has existed for about ten to twenty billion years. For scale, this can be compared to the four-and-a-half-billion-year age of the solar system, the time since the disappearance of the dinosaurs (sixty-five million years), and the age of the human race (about three million years).  
The event that marked the beginning of the universe was christened the Big Bang; the term has now entered the vernacular of our culture. Originally the name referred only to the single initiating event; now, however, astronomers have come to use it to mean the entire developmental process of the birth and expansion of the cosmos.  
The simple statement that the universe had a beginning in time is by now so obvious to astrophysicists that few give it a second thought. Yet it is a statement that has profound implications. Most civilizations embrace one of two opposite concepts of time. Linear time has a beginning, a duration, and an end; cyclical time, as its name suggests, continues around and around forever. In a universe that functions through cyclical time, the question of creation never arises; the universe always was and always will be. The minute you switch to linear time you immediately confront the vexing question not only of creation, but also of the Creator. Although there is no logical reason for the assumption, many people believe that if something comes into existence, it must do so in response to the actions of some rational being. Because of that belief, astronomers, even though they resist becoming involved in theological discussion, find themselves in one when they posit the Big Bang universe. It puts them squarely in the middle of an age-old debate.  
One common misconception about the Big Bang that should be disposed of immediately is the notion that the universal expansion is analogous to the explosion of an artillery shell. The galaxies are not like bits of shrapnel speeding away from a central explosion. The raisin-indough analogy is a more satisfactory way to think about the whole process.  
Directions: Read the above paragraph and answer the following:
Q.According to the passage, which of the following statements is NOT true?  
  • a)
    Many people believe that a rational impetus created the universe.  
  • b)
    The solar system was created immediately after the Big Bang.  
  • c)
    The universe is larger today than it was in the past.  
  • d)
    Different societies measure time differently.                  
  • e)
    Most cosmologists believe the universe to be 10 to 20 billion years old 
Correct answer is option 'B'. Can you explain this answer?

Parijat Menon answered
Explanation:

Incorrect Statement:
- The statement "The solar system was created immediately after the Big Bang" is not true according to the passage.

Reasoning:
- The passage mentions that the universe has existed for about ten to twenty billion years, while the solar system is about four-and-a-half-billion years old. This age gap indicates that the solar system was not created immediately after the Big Bang.
- The passage also discusses the concept of linear time and the beginning of the universe with the Big Bang event, which led to the expansion of the cosmos. This expansion does not imply that the solar system was created at the same time as the Big Bang.
Therefore, based on the information provided in the passage, the statement that the solar system was created immediately after the Big Bang is not true.

While many points are worth making in an evaluation of the single sixyear presidential term, one of the most telling points against the single term has not been advanced. This kind of constitutional limitation on elections is generally a product of systems with weak or non-existent political parties.
 Since there is no party continuity or corporate party integrity in such systems, there is no basis for putting trust in the desire for re-election as a safeguard against mismanagement in the executive branch. Better under those conditions to operate on the basis of negative assumptions against incumbents. I do not know if the earliest proposal for a single, nonrepeatable term was made in the 1820s because that was a period of severely weak political parties. But I do feel confident that this is a major reason, if not the only reason, that such a proposal has been popular since the 1940s.
 Though the association of the non-repeatable election with weak political parties is not in itself an argument against the limitation, the fallout from this association does contribute significantly to the negative argument. Single-term limitations are strongly associated with corruption. In any weak party system, including the presidential system, the onus of making deals and compromises, both shady and honourable, rests heavily upon individual candidates. Without some semblance of corporate integrity in a party, individual candidates have few opportunities to amortize their obligations across the spectrum of elective and appointive jobs and policy proposals.
The deals tend to be personalized and the payoffs come home to roost accordingly.  If that situation is already endemic in conditions of weak or nonexistent parties, adding to it the limitation against re-election means that candidates and officials, already prevented from amortizing their deals across space, are also unable to amortize their obligations temporally. This makes for a highly beleaguered situation. The single six-year term for presidents is an effort to compensate for the absence of a viable party system, but it is a compensation ultimately paid for by further weakening the party system itself.
 Observers, especially foreign observers, have often noted that one source of weakness in American political parties is the certainty of election every two or four years, not only because any artificial limitation on elections is a violation of democratic principles but also because when elections are set in a certain and unchangeable cycle, political parties do not have to remain alert but can disappear into inactivity until a known point prior to the next election. To rigidify matters by going beyond the determinacy of the electoral cycle to add an absolute rule of one term would hang still another millstone around the neck of already doddering political parties.  
Directions: Read the above paragraph and answer the following:
According to the passage, which of the following is most likely to be true of a political system with weak political parties? 
  • a)
    Politicians appoint unqualified people to important posts.  
  • b)
    Political parties favour frequent elections.  
  • c)
    Political bargains are made by individual candidates.  
  • d)
    Elections tend to occur with very great frequency.  
  • e)
    It encourages politicians to be more honest 
Correct answer is option 'C'. Can you explain this answer?

Ananya Sharma answered
Mapping the Passage
¶1 introduces the idea of a single presidential term.
¶2 argues that the single term is usually associated with countries with weak political parties and therefore popular when political parties are weak.
¶s3 and 4 argue that single-term systems encourage corruption.
¶5 argues that the single-term system is designed to make up for weak parties, but ends up making parties even weaker.
What does the author say about weak political parties? They lead to a preference for single terms, which, when enacted, lead to corruption. Evaluate the choices with this main chain of causes and effects in mind. While three choices aren‘t touched on by the author, (C) is the main point made in ¶3 and the beginning of ¶ 4: single-term systems encourage individual bargains (which ultimately lead to corruption.)

As formal organizations, business corporations are distinguished by their particular goals, which include maximization of profits, growth, and survival. Providing goods and services is a means to this end. If, for example, a number of individuals (outsiders or even insiders) believe that a company‘s aggressive marketing of infant formula in third world countries is morally wrong, the company is unlikely to be moved by arguments based on ethos alone as long as what it is doing remains profitable. But if those opposed to the company‘s practice organize a highly effective boycott of the company‘s products, their moral views will soon enter into the company‘s deliberations indirectly as limiting operating conditions. They can, at this point, no more be ignored than a prohibitive increase in the costs of certain raw materials.  
Although the concepts and categories of ethics may be applied to the conduct of corporations, there are important differences between the values and principles underlying corporate behaviour and those underlying the actions of most individuals. If corporations are by their nature end- or goal-directed how can they acknowledge acts as wrong in and of themselves? Is it possible to hold one criminally responsible for acts that if performed by a human person would result in criminal liability?  
The first case of this type to achieve widespread public attention was the attempt to prosecute the Ford Motor Company for manslaughter as the result of alleged negligent or reckless decision making concerning the safety engineering of the Pinto vehicle. Although the defendant corporation and its officers were found innocent after trial, the case can serve as an exemplar for our purposes.  
In essence, the prosecution in this case attempted to show that the corporation had produced and distributed a vehicle that was known to be defective at the time of production and sale, and that even after a great deal of additional information accumulated regarding the nature of the problems, the corporation took no action to correct them. The obvious non-corporate analogy would be the prosecution of a person who was driving a car with brakes known to be faulty, who does not have them repaired because it would cost too much, and who kills someone when the brakes eventually fail and the car does not stop in time. Such cases involving individuals are prosecuted and won regularly.
If corporations have no concept of right or wrong because they are exclusively goal-directed, can they be convicted in cases of this type, and what purpose would be served by such a conviction? Perhaps we can make a utilitarian argument for convicting corporations of such crimes. The argument would be that of deterrence; conviction and punishment would deter other corporations from taking similar actions under similar circumstances. However, there appears to be considerable evidence that deterrence does not work on corporations, even if, arguably, it works on individuals. The possibility of being discovered and the potential magnitude of the fine merely become more data to be included in the analysis of limiting conditions.  
Directions: Read the above paragraph and answer the following:
Q. If a company that produced shampoo products opted to stop the routine testing of its products on animals because it decided that it is wrong to cause the animals pain, what effect would this have on the argument made in the passage?  
  • a)
    It would strongly support the argument.  
  • b)
    It would substantially weaken the argument.  
  • c)
    It would neither support nor substantially weaken the argument.  
  • d)
    It would support the argument somewhat, but not conclusively.  
  • e)
    It would weaken the argument only if the company is a government owned company 
Correct answer is option 'D'. Can you explain this answer?

Rishika Sen answered
The correct answer is D. It would support the argument somewhat, but not conclusively.
The argument in the passage is that corporations are primarily goal-directed and focused on maximizing profits, growth, and survival. They are not inherently concerned with ethical considerations unless it directly affects their profitability.

If a company decides to stop routine animal testing because it believes it is wrong to cause animals pain, it does show that the company is considering ethical concerns. However, this decision alone does not conclusively prove that corporations can acknowledge acts as wrong in and of themselves. It could be argued that the company made this decision because it believes it will enhance its reputation and attract more customers who are concerned about animal welfare. Therefore, while it does support the argument to some extent, it does not provide conclusive evidence.

The passage below is accompanied by four questions. Based on the passage, choose the best answer for each question.
Over the past four centuries liberalism has been so successful that it has driven all its opponents off the battlefield. Now it is disintegrating, destroyed by a mix of hubris and internal contradictions, according to Patrick Deneen, a professor of politics at the University of Notre Dame. . . . Equality of opportunity has produced a new meritocratic aristocracy that has all the aloofness of the old aristocracy with none of its sense of noblesse oblige. Democracy has degenerated into a theatre of the absurd. And technological advances are reducing ever more areas of work into meaningless drudgery. “The gap between liberalism’s claims about itself and the lived reality of the citizenry” is now so wide that “the lie can no longer be accepted,” Mr Deneen writes. What better proof of this than the vision of 1,000 private planes whisking their occupants to Davos to discuss the question of “creating a shared future in a fragmented world”? . . .
Deneen does an impressive job of capturing the current mood of disillusionment, echoing leftwing complaints about rampant commercialism, right-wing complaints about narcissistic and bullying students, and general worries about atomisation and selfishness. But when he concludes that all this adds up to a failure of liberalism, is his argument convincing? . . . He argues that the essence of liberalism lies in freeing individuals from constraints. In fact, liberalism contains a wide range of intellectual traditions which provide different answers to the question of how to trade off the relative claims of rights and responsibilities, individual expression and social ties. . . . liberals experimented with a range of ideas from devolving power from the centre to creating national education systems.
Mr Deneen’s fixation on the essence of liberalism leads to the second big problem of his book: his failure to recognise liberalism’s ability to reform itself and address its internal problems. The late 19th century saw America suffering from many of the problems that are reappearing today, including the creation of a business aristocracy, the rise of vast companies, the corruption of politics and the sense that society was dividing into winners and losers. But a wide variety of reformers, working within the liberal tradition, tackled these problems head on. Theodore Roosevelt took on the trusts. Progressives cleaned up government corruption. University reformers modernised academic syllabuses and built ladders of opportunity. Rather than dying, liberalism reformed itself.
Mr Deneen is right to point out that the record of liberalism in recent years has been dismal. He is also right to assert that the world has much to learn from the premodern notions of liberty as self-mastery and self-denial. The biggest enemy of liberalism is not so much atomisation but old-fashioned greed, as members of the Davos elite pile their plates ever higher with perks and share options. But he is wrong to argue that the only way for people to liberate themselves from the contradictions of liberalism is “liberation from liberalism itself”. The best way to read “Why Liberalism Failed” is not as a funeral oration but as a call to action: up your game, or else.
All of the following statements are evidence of the decline of liberalism today, EXCEPT:
  • a)
    “And technological advances are reducing ever more areas of work into meaningless drudgery.”
  • b)
    “. . . the creation of a business aristocracy, the rise of vast companies . . .”
  • c)
    “Democracy has degenerated into a theatre of the absurd.”
  • d)
    “‘The gap between liberalism’s claims about itself and the lived reality of the citizenry’ is now so wide that ‘the lie can no longer be accepted,’. . .”
Correct answer is option 'A'. Can you explain this answer?

EduRev GMAT answered
  • All the options, other than A, are direct signs of declining or ineffective liberalism. 
  • Option B: Creation of business aristocracy, the author in the first paragraph says that liberalism promoted a meritocratic aristocracy and then went ahead to argue why the meritocratic aristocracy is not a good replacement of the old aristocracy. Creation of a business aristocracy and the rise of vast companies are against the ideals of liberalism. 
  • Option C: Democracy has degenerated into a theatre of the absurd, this clearly shows the non-functionality of liberalism and is a pretty valid argument for the decline of liberalism. 
  • Option D: The gap between liberalism’s claims about itself and the lived reality of the citizenry’ is now so wide that ‘the lie can no longer be accepted, this lines says that the gap between want liberalism asked us to do and what is actually different are two very different thing. This too can be an evidence of liberalism's decline. 
  • Option A: And technological advances are reducing ever more areas of work into meaningless drudgery, while this line does talk about the technological advancement in a negative sense, it does not necessarily provide evidence of the decline of liberalism per se. Instead, it highlights a potential consequence or critique within the context of technological advances. The negative impact of technology on certain types of work might be seen as a challenge that needs to be addressed within the liberal framework rather than direct evidence of the decline of liberalism.
  • The same challenge could be seen at a time when liberalism was prospering and thus is not an evidence of its decline. 

The ―paradox of tolerance‖ admonishes us that tolerance of the intolerant leads to intolerance. The Universal Declaration of Human Rights and the constitutions and laws of Western European democracies that adhere to the principle of freedom of speech all heed the warning of this conundrum and do not afford legal protection to extremist speech.  
While in Western European democracies, the speech of nondemocratic extremists has been successfully outlawed, in the United States the first amendment right to freedom of speech has been interpreted to encompass radical oration. The traditional justifications of this American stance originate in the belief that speech is entitled to greater tolerance than other kinds of activity. They are based on the belief that speech itself is valuable, and thus ascribe positive value to a very broad range of speech.
 According to the classical model, freedom of speech serves an indispensable function in the process of democratic self-government. From this perspective, the free speech principle need only protect political speech, comprised of all the facts, theories, and opinions relating to any issue on which the citizens must vote. Proponents of this view insist that even extremist views cannot be concealed from voting citizens, if these views bear on any public issue before them.
Protection of free speech serves the collective self-interests of a selfgoverning society made up of all rational, equal, and fully participating citizens who take their civic duties seriously. The fortress model is built on a foundation of pessimism, individualism, relativism, and self-doubt. At its deepest level, the fortress model values freedom of speech as a necessary precondition to the discovery and preservation of truth, but even at this level the function of speech remains primarily negative.  
From this perspective, the government and a majority of the people pose a great danger of intolerance. In spite of the high probability that their beliefs will eventually prove to be false, it is argued, people nonetheless tend to feel certain about them and, consequently, feel justified in requiring others to conform. Thus, the fortress model‘s prescription for combating the tendency to censor nonconforming views is to overprotect speech by providing a broad ―buffer zone‖ that encompasses extremist speech because its protection substantially diminishes the probability that inherently valuable speech will be suppressed.  
Directions: Read the above paragraph and answer the following
Q. The fortress model is ―built on a foundation of pessimism, individualism, relativism, and self-doubt.‖ Based on information in the passage, each of the following statements is a view held by those who believe in the fortress model of free speech EXCEPT:  
  • a)
    extremist political speech should be prohibited because it threatens democratic government.  
  • b)
    freedom of political speech is necessary in order to protect democratic government.  
  • c)
    a ban on extremist political speech raises the probability that more important political speech will also be banned.  
  • d)
    the government is unlikely to permit political speech that it finds objectionable unless the law prevents it from curbing political speech.  
  • e)
    the government should ensure that extremist speech, as long as it is political, is protected 
Correct answer is option 'A'. Can you explain this answer?

Explanation:

Statement: extremist political speech should be prohibited because it threatens democratic government.
- This statement aligns with the fortress model of free speech, as it suggests that extremist political speech can pose a threat to democratic government and therefore should be prohibited.

Statement: freedom of political speech is necessary in order to protect democratic government.
- This statement also reflects the beliefs of the fortress model, emphasizing the importance of protecting political speech in order to safeguard democratic government.

Statement: a ban on extremist political speech raises the probability that more important political speech will also be banned.
- This statement is consistent with the fortress model's view that overprotecting speech, including extremist political speech, can help prevent the suppression of valuable political speech.

Statement: the government is unlikely to permit political speech that it finds objectionable unless the law prevents it from curbing political speech.
- This statement highlights the fortress model's concern that without legal protections, the government may restrict political speech it deems objectionable.

Statement: the government should ensure that extremist speech, as long as it is political, is protected.
- While the fortress model advocates for broad protection of political speech, including extremist views, it does not specifically call for the protection of extremist political speech solely based on its political nature.
Therefore, the statement that does not align with the beliefs of the fortress model is: extremist political speech should be prohibited because it threatens democratic government (Option A).

The passage below is accompanied by four questions. Based on the passage, choose the best answer for each question.
Understanding romantic aesthetics is not a simple undertaking for reasons that are internal to the nature of the subject. Distinguished scholars, such as Arthur Lovejoy, Northrop Frye and Isaiah Berlin, have remarked on the notorious challenges facing any attempt to define romanticism. Lovejoy, for example, claimed that romanticism is “the scandal of literary history and criticism” . . . The main difficulty in studying the romantics, according to him, is the lack of any “single real entity, or type of entity” that the concept “romanticism” designates. Lovejoy concluded, “the word ‘romantic’ has come to mean so many things that, by itself, it means nothing” . . .
The more specific task of characterizing romantic aesthetics adds to these difficulties an air of paradox. Conventionally, “aesthetics” refers to a theory concerning beauty and art or the branch of philosophy that studies these topics. However, many of the romantics rejected the identification of aesthetics with a circumscribed domain of human life that is separated from the practical and theoretical domains of life. The most characteristic romantic commitment is to the idea that the character of art and beauty and of our engagement with them should shape all aspects of human life. Being fundamental to human existence, beauty and art should be a central ingredient not only in a philosophical or artistic life, but also in the lives of ordinary men and women. Another challenge for any attempt to characterize romantic aesthetics lies in the fact that most of the romantics were poets and artists whose views of art and beauty are, for the most part, to be found not in developed theoretical accounts, but in fragments, aphorisms and poems, which are often more elusive and suggestive than conclusive.
Nevertheless, in spite of these challenges the task of characterizing romantic aesthetics is neither impossible nor undesirable, as numerous thinkers responding to Lovejoy’s radical skepticism have noted. While warning against a reductive definition of romanticism, Berlin, for example, still heralded the need for a general characterization: “[Although] one does have a certain sympathy with Lovejoy’s despair…[he is] in this instance mistaken. There was a romantic movement…and it is important to discover what it is” . . .
Recent attempts to characterize romanticism and to stress its contemporary relevance follow this path. Instead of overlooking the undeniable differences between the variety of romanticisms of different nations that Lovejoy had stressed, such studies attempt to characterize romanticism, not in terms of a single definition, a specific time, or a specific place, but in terms of “particular philosophical questions and concerns” . . .
While the German, British and French romantics are all considered, the central protagonists in the following are the German romantics. Two reasons explain this focus: first, because it
has paved the way for the other romanticisms, German romanticism has a pride of place among the different national romanticisms . . . Second, the aesthetic outlook that was developed in Germany roughly between 1796 and 1801-02 — the period that corresponds to the heyday of what is known as “Early Romanticism” . . .— offers the most philosophical expression of romanticism since it is grounded primarily in the epistemological, metaphysical, ethical, and political concerns that the German romantics discerned in the aftermath of Kant’s philosophy.
According to the passage, recent studies on romanticism avoid “a single definition, a specific time, or a specific place” because they:
  • a)
    understand that the variety of romanticisms renders a general analysis impossible.
  • b)
    prefer to highlight the paradox of romantic aesthetics as a concept.
  • c)
    prefer to focus on the fundamental concerns of the romantics.
  • d)
    seek to discredit Lovejoy’s scepticism regarding romanticism.
Correct answer is option 'C'. Can you explain this answer?

Wizius Careers answered
Option C is the correct answer because it accurately reflects the passage's explanation of why recent studies on romanticism avoid seeking "a single definition, a specific time, or a specific place." According to the passage, these studies opt to characterize romanticism in terms of "particular philosophical questions and concerns" rather than attempting to provide a singular, all-encompassing definition. The reason for this approach is to delve into the fundamental concerns of the romantics, recognizing that romanticism is a complex and multifaceted movement with diverse expressions in different nations and contexts.
The passage suggests that romanticism is not easily confined to a single, universally applicable definition due to the variety of romanticisms in different nations. Instead of discrediting or refuting Lovejoy's skepticism (Option D), recent studies acknowledge the challenges and complexities of defining romanticism but seek to understand it by focusing on the core philosophical questions and concerns that were central to the romantics' worldview.

The passage below is accompanied by four questions. Based on the passage, choose the best answer for each question.
Understanding romantic aesthetics is not a simple undertaking for reasons that are internal to the nature of the subject. Distinguished scholars, such as Arthur Lovejoy, Northrop Frye and Isaiah Berlin, have remarked on the notorious challenges facing any attempt to define romanticism. Lovejoy, for example, claimed that romanticism is “the scandal of literary history and criticism” . . . The main difficulty in studying the romantics, according to him, is the lack of any “single real entity, or type of entity” that the concept “romanticism” designates. Lovejoy concluded, “the word ‘romantic’ has come to mean so many things that, by itself, it means nothing” . . .
The more specific task of characterizing romantic aesthetics adds to these difficulties an air of paradox. Conventionally, “aesthetics” refers to a theory concerning beauty and art or the branch of philosophy that studies these topics. However, many of the romantics rejected the identification of aesthetics with a circumscribed domain of human life that is separated from the practical and theoretical domains of life. The most characteristic romantic commitment is to the idea that the character of art and beauty and of our engagement with them should shape all aspects of human life. Being fundamental to human existence, beauty and art should be a central ingredient not only in a philosophical or artistic life, but also in the lives of ordinary men and women. Another challenge for any attempt to characterize romantic aesthetics lies in the fact that most of the romantics were poets and artists whose views of art and beauty are, for the most part, to be found not in developed theoretical accounts, but in fragments, aphorisms and poems, which are often more elusive and suggestive than conclusive.
Nevertheless, in spite of these challenges the task of characterizing romantic aesthetics is neither impossible nor undesirable, as numerous thinkers responding to Lovejoy’s radical skepticism have noted. While warning against a reductive definition of romanticism, Berlin, for example, still heralded the need for a general characterization: “[Although] one does have a certain sympathy with Lovejoy’s despair…[he is] in this instance mistaken. There was a romantic movement…and it is important to discover what it is” . . .
Recent attempts to characterize romanticism and to stress its contemporary relevance follow this path. Instead of overlooking the undeniable differences between the variety of romanticisms of different nations that Lovejoy had stressed, such studies attempt to characterize romanticism, not in terms of a single definition, a specific time, or a specific place, but in terms of “particular philosophical questions and concerns” . . .
While the German, British and French romantics are all considered, the central protagonists in the following are the German romantics. Two reasons explain this focus: first, because it
has paved the way for the other romanticisms, German romanticism has a pride of place among the different national romanticisms . . . Second, the aesthetic outlook that was developed in Germany roughly between 1796 and 1801-02 — the period that corresponds to the heyday of what is known as “Early Romanticism” . . .— offers the most philosophical expression of romanticism since it is grounded primarily in the epistemological, metaphysical, ethical, and political concerns that the German romantics discerned in the aftermath of Kant’s philosophy.
According to the romantics, aesthetics:
  • a)
    should be confined to a specific domain separate from the practical and theoretical aspects of life.
  • b)
    is primarily the concern of philosophers and artists, rather than of ordinary people.
  • c)
    is widely considered to be irrelevant to human existence.
  • d)
    permeates all aspects of human life, philosophical and mundane.
Correct answer is option 'D'. Can you explain this answer?

Wizius Careers answered
The most characteristic romantic commitment is to the idea that the character of art and beauty and of our engagement with them should shape all aspects of human life. Being fundamental to human existence, beauty and art should be a central ingredient not only in a philosophical or artistic life, but also in the lives of ordinary men and women.”
According to the passage, the romantics rejected the idea of confining aesthetics to a specific domain separate from practical and theoretical aspects of life. Instead, they believed that aesthetics, encompassing the character of art and beauty, should permeate all aspects of human existence, not only in philosophical or artistic lives but also in the lives of ordinary men and women.Therefore Option D is the correct answer.

Of course, in his attempts at field investigation, the historian is at the disadvantage that the countryside has changed in many respects since the period which he is studying. He is not permitted to use H.G. Wells‘s time machine, to enable him to see it as it actually was. Inevitably he is concerned in the main, if not exclusively, with literary and other materials, which have survived from that stretch of the past which interests him.
 Old maps may be plans of cities, charts of sea coasts and estuaries, cartularies of landed estates, or topographic delineations of land areas. These clearly engage the interest of historians and geographers alike, and they call for a combination of the methods and viewpoints of each. Maps can be conceived of and considered in several quite different ways, being properly regarded, and so assessed, as works of art—at best as objects of colour, skill, form, and beauty. They may alternatively be regarded purely for their cartographic aesthetic.  
The main queries which then arise are the following: how is it that the map-maker has carried out his task and with skill of what echelon and with what degree of success has he done so? Such an inquiry falls to the specialist field of historical cartography. An antiquarian map may also be approached in a means akin to that of the student who conceives it as a font contemporaneous with the time of its production. Thus, the historical cartographer may seek to bring grist to his mill and to consider the map‘s reliability as a satisfactory source of empirical evidence. By such means also the regional historian, in his search for essentials about such past matters as the availability of roads, the extent of enclosed farmland, or the number and location of mines and quarries, is no less an interested party.
 The value of old maps as documents useful for historicity depends necessarily on to what degree they depict and on how accurately. For virtually all periods of pre-modern history some maps have survived to serve as historiography, depicting, however imperfectly, certain features of past geography. The work of Claudius Ptolemy—who lived in the 2nd century A.D.—for centuries provided the basis for maps of the known world and its major regions. Although many were drawn on the scientific basis which he provided, they nevertheless embodied many errors—of location, distance, and the shape of areas of land and sea.  
The medieval portolan charts of the Mediterranean Sea and the later charts which provided sailing directions, produced in Holland, were accurate enough to be useful in practical navigation. Plans of important cities of Europe, so well-drawn as to yield evidence of their earlier form and extent, are notably offered in Braun and Hogenberg‘s Civitates Orbis Terrarum, published at Cologne and, in England, in John Speed‘s plans of cities. Similarly, John Ogilby‘s Britannia, Volume the First, appearing in 1675, gives detailed information of England's road system as it existed nearly three centuries ago. However, few of the early maps approach modern standards, which require accurate representation of distances and of heights above mean sea-level and the use of carefully distinguished symbols. This is because it was not until the 18th century that cartography, as an exact science, was born.  
Directions: Read the above paragraph and answer the following:
Q.With which of the following statements would the author be most likely to agree?  
  • a)
    Old maps provide important information about the past, even if they are somewhat misleading.  
  • b)
    Modern maps, in general, are more accurate than maps produced in the 18th century.  
  • c)
    The maps in Braun and Hogenberg‘s book have no historical value because of their errors.  
  • d)
    Claudius Ptolemy‘s maps were the most accurate ever made prior to the birth of modern cartography.                  
  • e)
    The field of cartography is on a downward spiral
Correct answer is option 'A'. Can you explain this answer?

Sakshi Das answered
Explanation:

Importance of Old Maps:
Old maps provide important information about the past, even if they are somewhat misleading. They serve as valuable historical documents that offer insights into the geography, topography, and features of earlier times.

Limited Accuracy:
While old maps may contain errors or inaccuracies, they still hold significance in understanding past landscapes and civilizations. Despite their limitations, they provide a glimpse into how people perceived and navigated the world in earlier centuries.

Historical Value:
Maps like those found in Braun and Hogenberg's Civitates Orbis Terrarum or John Speed's plans of cities may contain errors, but they are still valuable for studying the historical development of urban areas and landscapes.

Evolution of Cartography:
The author acknowledges that modern maps are more accurate than those produced in earlier centuries. However, the historical value of old maps lies in their ability to shed light on the past, even if they are not as precise as contemporary cartographic representations.

Henry Varnum Poor, editor of American Railroad Journal, drew the important elements of the image of the railroad together in 1851, ―Look at the results of this material progress...the vigor, life, and executive energy that followed in its train, rapidly succeeded by wealth, the refinement and intellectual culture of a high civilization. All this is typified, in a degree, by a locomotive. The combination in its construction of nice art and scientific application of power, its speed surpassing that of our proudest courser, and its immense strength, are all characteristic of our age and tendencies. To us, like the telegraph, it is essential, it constitutes a part of our nature, is a condition of our being what we are.
In thei third decade of the nineteenth century, Americans began to define their character in light of the new railroads. They liked the idea that it took special people to foresee and capitalize on the promise of science. Railroad promoters, using the steam engine as a metaphor for what they thought Americans were and what they thought Americans were becoming, frequently discussed parallels between the locomotive and national character, pointing out that both possessed youth, power, speed, single-mindedness, and bright prospects. 
Poor was, of course, promoting acceptance of railroads and enticing his readers to open their pocketbooks. But his metaphors had their dark side. A locomotive was quite unlike anything Americans had ever seen. It was large, mysterious and dangerous; many thought that it was a monster waiting to devour the unwary. There was a suspicion that a country founded upon Jeffersonian agrarian principles had bought a ticket and boarded a train pulled by some iron monster into the dark recesses of an unknown future.
To ease such public apprehensions, promoters, poets, editors, and writers alike adopted the notion that locomotives were really only ―iron horses,‖ an early metaphor that lingered because it made steam technology ordinary and understandable. Iron horse metaphors assuaged fears about inherent defects in the national character, prompting images of a more secure future, and made an alien technology less frightening, and even comforting and congenial.
Essayist Ralph Waldo Emerson saw the locomotive as an agent of domestic harmony. He observed that ―the locomotive and the steamboat, like enormous shuttles, shoot every day across the thousand various threads of national descent and employment and bind them fast in one web,"adding ―an hourly assimilation goes forward, and there is no danger that local peculiarities and hostilities should be preserved. To us Americans, it seems to have fallen as a political aid. We could not else have held the vast North America together, which we now engage to do."
Direction: Read the above Paragraph and answer the follownig Quetions
Q.  According to the passage, which of the following is most likely to be true about Ralph Waldo Emerson‘s beliefs? 
  • a)
    He felt that Americans should adhere strictly to Jeffersonian agrarian principles.  
  • b)
    He thought that the railroad was as important as the telegraph. 
  • c)
    He felt that technological progress would help to unify Americans.  
  • d)
    He thought that railroad promoters were acting against America‘s best interests. 
  • e)
    His metaphors had a dark side to them 
Correct answer is option 'C'. Can you explain this answer?

Rashi Shah answered
Mapping the Passage:
¶1 describes the opinions of one railroad promoter (Poor), who tied the railroad to the progressive nature of American character.
¶2 describes the American idea of the time that the railroad reflected elements of American character.
¶s3 and 4 discuss the fears associated with the railroad and the metaphors presented to counter them.
¶5 describes the way that Americans were won over to the railroad by these metaphors (Emerson).
Where is Emerson mentioned? Review the last paragraph: Emerson thought that the locomotive kept the nation together. Look for an answer choice that ties into this unity: (C) does just that.

Once surrounded and protected by vast wilderness, many of the national parks are adversely affected by activities outside their boundaries. The National Park Organic Act established the national park system and empowered the Secretary of the Interior to manage activities within the parks. Conditions outside park boundaries are not subject to regulation by the Park Service unless they involve the direct use of park resources.
 Several approaches to protecting the national parks from external degradation have been proposed, such as one focusing on enacting federal legislation granting the National Park Service broader powers over lands adjacent to the national parks. Legislation addressing external threats to the national parks twice passed the House of Representatives but died without action in the Senate. Also brought to the table as a possible remedy is giving the states bordering the parks a significant and meaningful role in developing federal park management policy.  
Because the livelihood of many citizens is linked to the management of national parks, local politicians often encourage state involvement in federal planning. But, state legislatures have not always addressed the fundamental policy issues of whether states should protect park wildlife.  
Timber harvesting, ranching and energy exploration compete with wildlife within the local ecosystem. Priorities among different land uses are not generally established by current legislation. Additionally, often no mechanism exists to coordinate planning by the state environmental regulatory agencies. These factors limit the impact of legislation aimed at protecting park wildlife and the larger park ecosystem. 
Even if these deficiencies can be overcome, state participation must be consistent with existing federal legislation. States lack jurisdiction within national parks themselves, and therefore state solutions cannot reach activities inside the parks, thus limiting state action to the land adjacent to the national parks. Under the supremacy clause, federal laws and regulations supersede state action if state law conflicts with federal legislation, if Congress precludes local regulation, or if federal regulation is so pervasive that no room remains for state control. Assuming that federal regulations leave open the possibility of state control, state participation in policy making must be harmonized with existing federal legislation.  
The residents of states bordering national parks are affected by park management policies. They in turn affect the success of those policies. This interrelationship must be considered in responding to the external threats problem. Local participation is necessary in deciding how to protect park wildlife. Local interests should not, however, dictate national policy, nor should they be used as a pretext to ignore the threats to park regions.
Direction: Read the above Paragraph and answer the follownig Quetions
Q.The passage provides support for which of the following assertions? 
  • a)
    The National Park Organic Act gave the Secretary of the Interior the right to overrule state government policy in lands adjacent to national parks.  
  • b)
    The federal government has been selling national park land to state governments in order to raise money for wildlife conservation.  
  • c)
    The actions of state governments have often failed to promote the interests of national park wildlife.  
  • d)
    Local politicians want the federal government to turn control of national parks over to state governments.
  • e)
    Timber harvesting and energy exploration have not had any impact on national parks 
Correct answer is option 'C'. Can you explain this answer?

Mapping the Passage:
¶1 describes a problem facing national parks: negative effects from the land
surrounding them.
¶2 describes one approach to dealing with the problem: federal legislation, which
failed.
¶3 and 4 describe a second approach: giving power to states to cooperate with
adjacent national parks, and describe the problems with it.
¶5 argues that state participation must be tied to federal regulations.
¶6 argues that any solution requires a national response with elements of local participation.
An Inference question, this one requires students to find that one option which can logically follow from the information in the passage without making any extreme asumptions. Only (C) has support in the passage. The claim is originally made in lines 17-20, and ¶s 4 and 5 offer support.

Once surrounded and protected by vast wilderness, many of the national parks are adversely affected by activities outside their boundaries. The National Park Organic Act established the national park system and empowered the Secretary of the Interior to manage activities within the parks. Conditions outside park boundaries are not subject to regulation by the Park Service unless they involve the direct use of park resources.
Several approaches to protecting the national parks from external degradation have been proposed, such as one focusing on enacting federal legislation granting the National Park Service broader powers over lands adjacent to the national parks. Legislation addressing external threats to the national parks twice passed the House of Representatives but died without action in the Senate. Also brought to the table as a possible remedy is giving the states bordering the parks a significant and meaningful role in developing federal park management policy.
Because the livelihood of many citizens is linked to the management of national parks, local politicians often encourage state involvement in federal planning. But, state legislatures have not always addressed the fundamental policy issues of whether states should protect park wildlife.
Timber harvesting, ranching and energy exploration compete with wildlife within the local ecosystem. Priorities among different land uses are not generally established by current legislation. Additionally, often no mechanism exists to coordinate planning by the state environmental regulatory agencies. These factors limit the impact of legislation aimed at protecting park wildlife and the larger park ecosystem.
Even if these deficiencies can be overcome, state participation must be consistent with existing federal legislation. States lack jurisdiction within national parks themselves, and therefore state solutions cannot reach activities inside the parks, thus limiting state action to the land adjacent to the national parks. Under the supremacy clause, federal laws and regulations supersede state action if state law conflicts with federal legislation, if Congress precludes local regulation, or if federal regulation is so pervasive that no room remains for state control. Assuming that federal regulations leave open the possibility of state control, state participation in policy making must be harmonized with existing federal legislation.
The residents of states bordering national parks are affected by park management policies. They in turn affect the success of those policies. This interrelationship must be considered in responding to the external threats problem. Local participation is necessary in deciding how to protect park wildlife. Local interests should not, however, dictate national policy, nor should they be used as a pretext to ignore the threats to park regions. 
Direction: Read the above Paragraph and answer the following Questions:
In the context of the passage, the phrase external degradation (lines 8-9) refers to which of the following:  
  • a)
    Threats to national parks arising from the House of Representative's willingness to address environmental issues. 
  • b)
    Threats to national parks arising from local politicians‘ calls for greater state involvement in national park planning.
  • c)
    Threats to national parks arising from state government environmental policies. 
  • d)
    Threats to national parks arising from the National Park Organic Act. 
  • e)
    Threats to national parks arising from the lack of local support. 
Correct answer is option 'B'. Can you explain this answer?

Pooja Dasgupta answered
The passage discusses how activities outside the boundaries of national parks can negatively impact the parks. The phrase "external degradation" refers to these threats, which are influenced by local politicians advocating for state involvement in federal park planning. This suggests that the threats come from actions and decisions made at the state level, rather than from federal legislation or lack of local support.

Henry Varnum Poor, editor of American Railroad Journal, drew the important elements of the image of the railroad together in 1851, ―Look at the results of this material progress...the vigor, life, and executive energy that followed in its train, rapidly succeeded by wealth, the refinement and intellectual culture of a high civilization. All this is typified, in a degree, by a locomotive. The combination in its construction of nice art and scientific application of power, its speed surpassing that of our proudest courser, and its immense strength, are all characteristic of our age and tendencies. To us, like the telegraph, it is essential, it constitutes a part of our nature, is a condition of our being what we are.
In the third decade of the nineteenth century, Americans began to define their character in light of the new railroads. They liked the idea that it took special people to foresee and capitalize on the promise of science. Railroad promoters, using the steam engine as a metaphor for what they thought Americans were and what they thought Americans were becoming, frequently discussed parallels between the locomotive and national character, pointing out that both possessed youth, power, speed, single-mindedness, and bright prospects.
Poor was, of course, promoting acceptance of railroads and enticing his readers to open their pocketbooks. But his metaphors had their dark side. A locomotive was quite unlike anything Americans had ever seen. It was large, mysterious and dangerous; many thought that it was a monster waiting to devour the unwary. There was a suspicion that a country founded upon Jeffersonian agrarian principles had bought a ticket and boarded a train pulled by some iron monster into the dark recesses of an unknown future.  
To ease such public apprehensions, promoters, poets, editors, and writers alike adopted the notion that locomotives were really only ―iron horses, an early metaphor that lingered because it made steam technology ordinary and understandable. Iron horse metaphors assuaged fears about inherent defects in the national character, prompting images of a more secure future, and made an alien technology less frightening, and even comforting and congenial.
Essayist Ralph Waldo Emerson saw the locomotive as an agent of domestic harmony. He observed that ―the locomotive and the steamboat, like enormous shuttles, shoot every day across the thousand various threads of national descent and employment and bind them fast in one web, adding ―an hourly assimilation goes forward, and there is no danger that local peculiarities and hostilities should be preserved. To us Americans, it seems to have fallen as a political aid. We could not else have held the vast North America together, which we now engage to do.
Direction: Read the above Paragraph and answer the follownig Quetions
Q.  Which of the following claims would the author of the passage most agree with? 
  • a)
    The railroad undermined America‘s progressive tendencies. 
  • b)
    Railroad promoters like Poor denounced Jeffersonian agrarian principles.  
  • c)
    The Ameicans in general were against the railroad 
  • d)
    Ralph Waldo Emerson thought that the railroad would harm America. 
  • e)
    Americans generally supported the development of the railroad. 
Correct answer is option 'E'. Can you explain this answer?

Kajal Kulkarni answered
Mapping of Paragraph
¶1 describes the opinions of one railroad promoter (Poor), who tied the railroad to the
progressive nature of American character.
¶2 describes the American idea of the time that the railroad reflected elements of
American character.
¶s3 and 4 discuss the fears associated with the railroad and the metaphors presented
to counter them.
¶5 describes the way that Americans were won over to the railroad by these
metaphors (Emerson).
The question stem gives you a big hint—take the statement "at face value" and "objectively." Don't over think! The passage itself is straightforward, so review the author‘s main gist: the railroad reflected American character at the time, and despite a few misgivings, American were generally on board. While three answer choices don‘t fit with what the author argues, (E) fits and is supported extensively
in the last paragraph.

The passage below is accompanied by four questions. Based on the passage, choose the best answer for each question.
The Second Hand September campaign, led by Oxfam . . . seeks to encourage shopping at local organisations and charities as alternatives to fast fashion brands such as Primark and Boohoo in the name of saving our planet. As innocent as mindless scrolling through online shops may seem, such consumers are unintentionally—or perhaps even knowingly —contributing to an industry that uses more energy than aviation. . . .
Brits buy more garments than any other country in Europe, so it comes as no shock that many of those clothes end up in UK landfills each year: 300,000 tonnes of them, to be exact. This waste of clothing is destructive to our planet, releasing greenhouse gasses as clothes are burnt as well as bleeding toxins and dyes into the surrounding soil and water. As ecologist Chelsea Rochman bluntly put it, “The mismanagement of our waste has even come back to haunt us on our dinner plate.”
It’s not surprising, then, that people are scrambling for a solution, the most common of which is second-hand shopping. Retailers selling consigned clothing are currently expanding at a rapid rate . . . If everyone bought just one used item in a year, it would save 449 million lbs of waste, equivalent to the weight of 1 million Polar bears. “Thrifting” has increasingly become a trendy practice. London is home to many second-hand, or more commonly coined ‘vintage’, shops across the city from Bayswater to Brixton.
So you’re cool and you care about the planet; you’ve killed two birds with one stone. But do people simply purchase a second-hand item, flash it on Instagram with #vintage and call it a day without considering whether what they are doing is actually effective?
According to a study commissioned by Patagonia, for instance, older clothes shed more microfibres. These can end up in our rivers and seas after just one wash due to the worn material, thus contributing to microfibre pollution. To break it down, the amount of microfibres released by laundering 100,000 fleece jackets is equivalent to as many as 11,900 plastic grocery bags, and up to 40 per cent of that ends up in our oceans. . . . So where does this leave second-hand consumers? [They would be well advised to buy] high-quality items that shed less and last longer [as this] combats both microfibre pollution and excess garments ending up in landfills. . . .
Luxury brands would rather not circulate their latest season stock around the globe to be sold at a cheaper price, which is why companies like ThredUP, a US fashion resale marketplace, have not yet caught on in the UK. There will always be a market for consignment but there is also a whole generation of people who have been taught that only buying new products is the norm; second-hand luxury goods are not in their psyche. Ben Whitaker, director at Liquidation Firm B-Stock, told Prospect that unless recycling becomes cost-effective and filters into mass production, with the right technology to partner it, “high-end retailers would rather put brand before sustainability.”
The central idea of the passage would be undermined if:
  • a)
    Customers bought all their clothes online.
  • b)
    Clothes were not thrown and burnt in landfills
  • c)
    Second-hand stores sold only high-quality clothes.
  • d)
    Primark and Boohoo recycled their clothes for vintage stores
Correct answer is option 'C'. Can you explain this answer?

EduRev GMAT answered
  • The central idea of the passage is the promotion of sustainable shopping practices, particularly second-hand shopping, as a means to combat the detrimental environmental effects of the fashion industry. But, the passage also discusses the need for consumers to be mindful of the environmental impact of their clothing choices, opting for high-quality items that last longer and shed fewer microfibers.
  • The passage argues that opting for second clothing might not always be beneficial for the environment by highlighting the microfibre pollution that they can potentially cause. Now, if the second-hand clothes being sold were only of higher quality, it would take care of this problem ([They would be well advised to buy] high-quality items that shed less and last longer [as this] combats both microfibre pollution and excess garments ending up in landfills”)
  • So, the correct answer is Option C.
  • Option A is more about the purchasing channel than the nature of the clothes so it does not necessarily undermine the central idea of the passage.
  • Option B supports the central idea by reducing environmental harm.
  • Option D could align with the sustainability goal and support the central idea, so it doesn't necessarily undermine it.

In public Greek life, a man had to make his way at every step through the immediate persuasion of the spoken word. Whether it be addressing an assembly, a law-court or a more restricted body, his oratory would be a public affair rather than under the purview of a quiet committee, without the support of circulated commentary, and with no backcloth of daily reportage to make his own or others‘ views familiar to his hearers. The oratory's immediate effect was all-important; it would be naive to expect that mere reasonableness or an inherently good case would equate to a satisfactory appeal. Therefore, it was early realized that persuasion was an art, up to a point teachable, and a variety of specific pedagogy was well established in the second half of the fifth century. When the sophists claimed to teach their pupils how to succeed in public life, rhetoric was a large part of what they meant, though, to do them justice, it was not the whole.
Skill naturally bred mistrust. If a man of good will had need of expression advanced of mere twaddle, to learn how to expound his contention effectively, the truculent or pugnacious could be taught to dress their case in well-seeming guise. It was a standing charge against the sophists that they 'made the worse appear the better cause,‘ and it was this immoral lesson which the hero of Aristophanes‘ Clouds went to learn from, of all people, Socrates. Again, the charge is often made in court that the opponent is an adroit orator and the jury must be circumspect so as not to let him delude them. From the frequency with which this crops up, it is patent that the accusation of cleverness might damage a man. In Greece, juries, of course, were familiar with the style, and would recognize the more evident artifices, but it was worth a litigant‘s while to get his speech written for him by an expert. Persuasive oratory was certainly one of the pressures that would be effective in an Athenian law-court.
A more insidious danger was the inevitable desire to display this art as an art. It is not easy to define the point at which a legitimate concern with style shades off into preoccupation with manner at the expense of matter, but it is easy to perceive that many Greek writers of the fourth and later centuries passed that danger point. The most influential was Isocrates, who polished for long years his pamphlets, written in the form of speeches, and taught to many pupils the smooth and easy periods he had perfected. Isocrates took to the written word in compensation for his inadequacy in live oratory; the tough and nervous tones of a Demosthenes were far removed from his, though they, too, were based on study and practice. The exaltation of virtuosity did palpable harm. The balance was always delicate, between style as a vehicle and style as an end in itself.
 We must not try to pinpoint a specific moment when it, once and for all, tipped over; but certainly, as time went on, virtuosity weighed heavier. While Greek freedom lasted, and it mattered what course of action a Greek city decided to take, rhetoric was a necessary preparation for public life, whatever its side effects. It had been a source of strength for Greek civilization that its problems, of all kinds, were thrashed out very much in public. The shallowness which the study of rhetoric might (not must) encourage was the corresponding weakness.  
Directions: Read the above paragraph and answer the following
Q.Implicit in the statement that the exaltation of virtuosity was not due mainly to Isocrates because public display was normal in a world that talked far more than it read is the assumption that:  
  • a)
    Isocrates was actually concerned as much with the content of his speeches as with their style.  
  • b)
    excessive concern with style is bound to arise in a world dominated by public display.  
  • c)
    the Greeks were guilty of exalting virtuosity in their public art and architecture as well.  
  • d)
    Isocrates was less influential than previous historians estimated.        
  • e)
    there should be no connection between communication style and public display of thoughts 
Correct answer is option 'B'. Can you explain this answer?

Dhruba Jain answered
Mapping the Passage:
¶1 introduces the importance of rhetoric in Greek life and the fact that it was taught.
¶2 explains why rhetorical skill was sometimes mistrusted, but still sought after.
¶3 states that as rhetoric (in writing and speech) became more of an art, its original purpose was corrupted.
¶4 states that the Greek system required rhetorical skills and therefore inherited rhetoric's drawbacks as well.
Make sure that you untangle tough questions, paraphrasing what‘s being asked, before trying to answer them. What paragraph is being discussed? ¶3, the argument that the art of rhetoric became too important. The question stem just says that this happened because the culture was concerned with public display. Assumptions bridge gaps in reasoning. Here, it would connect art and public display. Only (B) and (C) deal with both of these concepts. If (B) is true, we have a valid explanation for why art became so important in this particular culture. If it‘s not true, there‘s no reason why they should be connected, and the author‘s argument falls apart. (B) has to be a valid assumption

The passage below is accompanied by four questions. Based on the passage, choose the best answer for each question.
Understanding romantic aesthetics is not a simple undertaking for reasons that are internal to the nature of the subject. Distinguished scholars, such as Arthur Lovejoy, Northrop Frye and Isaiah Berlin, have remarked on the notorious challenges facing any attempt to define romanticism. Lovejoy, for example, claimed that romanticism is “the scandal of literary history and criticism” . . . The main difficulty in studying the romantics, according to him, is the lack of any “single real entity, or type of entity” that the concept “romanticism” designates. Lovejoy concluded, “the word ‘romantic’ has come to mean so many things that, by itself, it means nothing” . . .
The more specific task of characterizing romantic aesthetics adds to these difficulties an air of paradox. Conventionally, “aesthetics” refers to a theory concerning beauty and art or the branch of philosophy that studies these topics. However, many of the romantics rejected the identification of aesthetics with a circumscribed domain of human life that is separated from the practical and theoretical domains of life. The most characteristic romantic commitment is to the idea that the character of art and beauty and of our engagement with them should shape all aspects of human life. Being fundamental to human existence, beauty and art should be a central ingredient not only in a philosophical or artistic life, but also in the lives of ordinary men and women. Another challenge for any attempt to characterize romantic aesthetics lies in the fact that most of the romantics were poets and artists whose views of art and beauty are, for the most part, to be found not in developed theoretical accounts, but in fragments, aphorisms and poems, which are often more elusive and suggestive than conclusive.
Nevertheless, in spite of these challenges the task of characterizing romantic aesthetics is neither impossible nor undesirable, as numerous thinkers responding to Lovejoy’s radical skepticism have noted. While warning against a reductive definition of romanticism, Berlin, for example, still heralded the need for a general characterization: “[Although] one does have a certain sympathy with Lovejoy’s despair…[he is] in this instance mistaken. There was a romantic movement…and it is important to discover what it is” . . .
Recent attempts to characterize romanticism and to stress its contemporary relevance follow this path. Instead of overlooking the undeniable differences between the variety of romanticisms of different nations that Lovejoy had stressed, such studies attempt to characterize romanticism, not in terms of a single definition, a specific time, or a specific place, but in terms of “particular philosophical questions and concerns” . . .
While the German, British and French romantics are all considered, the central protagonists in the following are the German romantics. Two reasons explain this focus: first, because it
has paved the way for the other romanticisms, German romanticism has a pride of place among the different national romanticisms . . . Second, the aesthetic outlook that was developed in Germany roughly between 1796 and 1801-02 — the period that corresponds to the heyday of what is known as “Early Romanticism” . . .— offers the most philosophical expression of romanticism since it is grounded primarily in the epistemological, metaphysical, ethical, and political concerns that the German romantics discerned in the aftermath of Kant’s philosophy.
The main difficulty in studying romanticism is the:
  • a)
    elusive and suggestive nature of romantic aesthetics.
  • b)
    lack of clear conceptual contours of the domain.
  • c)
    controversial and scandalous history of romantic literature.
  • d)
    absence of written accounts by romantic poets and artists.
Correct answer is option 'B'. Can you explain this answer?

EduRev GMAT answered
The main difficulty in studying the romantics, according to him, is the lack of any “single real entity, or type of entity” that the concept “romanticism” designates.”
Option B is the correct answer because it accurately captures the main difficulty highlighted in the passage when studying romanticism. The passage, particularly referencing Arthur Lovejoy, emphasizes the challenge posed by the lack of clear conceptual contours or a single, cohesive entity associated with the term "romanticism." Lovejoy's assertion that romanticism is the "scandal of literary history and criticism" underscores the difficulty in defining the boundaries and characteristics of this literary and artistic movement.
The elusive and suggestive nature of romantic aesthetics (Option A) is mentioned in the passage as a challenge, but it is not identified as the main difficulty. Similarly, the controversial and scandalous history of romantic literature (Option C) is not specified as the primary obstacle. The absence of written accounts by romantic poets and artists (Option D) is acknowledged as a challenge, but the primary difficulty highlighted in the passage is the lack of clear conceptual contours associated with romanticism.

The passage below is accompanied by four questions. Based on the passage, choose the best answer for each question.
The Second Hand September campaign, led by Oxfam . . . seeks to encourage shopping at local organisations and charities as alternatives to fast fashion brands such as Primark and Boohoo in the name of saving our planet. As innocent as mindless scrolling through online shops may seem, such consumers are unintentionally—or perhaps even knowingly —contributing to an industry that uses more energy than aviation. . . .
Brits buy more garments than any other country in Europe, so it comes as no shock that many of those clothes end up in UK landfills each year: 300,000 tonnes of them, to be exact. This waste of clothing is destructive to our planet, releasing greenhouse gasses as clothes are burnt as well as bleeding toxins and dyes into the surrounding soil and water. As ecologist Chelsea Rochman bluntly put it, “The mismanagement of our waste has even come back to haunt us on our dinner plate.”
It’s not surprising, then, that people are scrambling for a solution, the most common of which is second-hand shopping. Retailers selling consigned clothing are currently expanding at a rapid rate . . . If everyone bought just one used item in a year, it would save 449 million lbs of waste, equivalent to the weight of 1 million Polar bears. “Thrifting” has increasingly become a trendy practice. London is home to many second-hand, or more commonly coined ‘vintage’, shops across the city from Bayswater to Brixton.
So you’re cool and you care about the planet; you’ve killed two birds with one stone. But do people simply purchase a second-hand item, flash it on Instagram with #vintage and call it a day without considering whether what they are doing is actually effective?
According to a study commissioned by Patagonia, for instance, older clothes shed more microfibres. These can end up in our rivers and seas after just one wash due to the worn material, thus contributing to microfibre pollution. To break it down, the amount of microfibres released by laundering 100,000 fleece jackets is equivalent to as many as 11,900 plastic grocery bags, and up to 40 per cent of that ends up in our oceans. . . . So where does this leave second-hand consumers? [They would be well advised to buy] high-quality items that shed less and last longer [as this] combats both microfibre pollution and excess garments ending up in landfills. . . .
Luxury brands would rather not circulate their latest season stock around the globe to be sold at a cheaper price, which is why companies like ThredUP, a US fashion resale marketplace, have not yet caught on in the UK. There will always be a market for consignment but there is also a whole generation of people who have been taught that only buying new products is the norm; second-hand luxury goods are not in their psyche. Ben Whitaker, director at Liquidation Firm B-Stock, told Prospect that unless recycling becomes cost-effective and filters into mass production, with the right technology to partner it, “high-end retailers would rather put brand before sustainability.”
The act of “thrifting”, as described in the passage, can be considered ironic because it:
  • a)
    Has created environmental problems.
  • b)
    Is not cost-effective for retailers
  • c)
    Offers luxury clothing at cut-rate prices.
  • d)
    Is an anti-consumerist attitude.
Correct answer is option 'A'. Can you explain this answer?

EduRev GMAT answered
The irony of "thrifting," as discussed in the passage, is rooted in its unintended environmental consequences. While thrifting is commonly associated with sustainable and eco-friendly practices, the passage highlights a potential environmental issue linked to the act. Specifically, the passage mentions a study commissioned by Patagonia that reveals older clothes, often found in second-hand stores, tend to shed more microfibers. These microfibers can end up in rivers and oceans, contributing to microfiber pollution. Therefore, the seemingly environmentally conscious act of thrifting, aimed at reducing waste, may inadvertently result in environmental problems through the shedding of microfibers during the washing of older garments.Therefore Option A is the correct answer

The passage below is accompanied by four questions. Based on the passage, choose the best answer for each question.
Over the past four centuries liberalism has been so successful that it has driven all its opponents off the battlefield. Now it is disintegrating, destroyed by a mix of hubris and internal contradictions, according to Patrick Deneen, a professor of politics at the University of Notre Dame. . . . Equality of opportunity has produced a new meritocratic aristocracy that has all the aloofness of the old aristocracy with none of its sense of noblesse oblige. Democracy has degenerated into a theatre of the absurd. And technological advances are reducing ever more areas of work into meaningless drudgery. “The gap between liberalism’s claims about itself and the lived reality of the citizenry” is now so wide that “the lie can no longer be accepted,” Mr Deneen writes. What better proof of this than the vision of 1,000 private planes whisking their occupants to Davos to discuss the question of “creating a shared future in a fragmented world”? . . .
Deneen does an impressive job of capturing the current mood of disillusionment, echoing leftwing complaints about rampant commercialism, right-wing complaints about narcissistic and bullying students, and general worries about atomisation and selfishness. But when he concludes that all this adds up to a failure of liberalism, is his argument convincing? . . . He argues that the essence of liberalism lies in freeing individuals from constraints. In fact, liberalism contains a wide range of intellectual traditions which provide different answers to the question of how to trade off the relative claims of rights and responsibilities, individual expression and social ties. . . . liberals experimented with a range of ideas from devolving power from the centre to creating national education systems.
Mr Deneen’s fixation on the essence of liberalism leads to the second big problem of his book: his failure to recognise liberalism’s ability to reform itself and address its internal problems. The late 19th century saw America suffering from many of the problems that are reappearing today, including the creation of a business aristocracy, the rise of vast companies, the corruption of politics and the sense that society was dividing into winners and losers. But a wide variety of reformers, working within the liberal tradition, tackled these problems head on. Theodore Roosevelt took on the trusts. Progressives cleaned up government corruption. University reformers modernised academic syllabuses and built ladders of opportunity. Rather than dying, liberalism reformed itself.
Mr Deneen is right to point out that the record of liberalism in recent years has been dismal. He is also right to assert that the world has much to learn from the premodern notions of liberty as self-mastery and self-denial. The biggest enemy of liberalism is not so much atomisation but old-fashioned greed, as members of the Davos elite pile their plates ever higher with perks and share options. But he is wrong to argue that the only way for people to liberate themselves from the contradictions of liberalism is “liberation from liberalism itself”. The best way to read “Why Liberalism Failed” is not as a funeral oration but as a call to action: up your game, or else.
The author of the passage refers to “the Davos elite” to illustrate his views on:
  • a)
    The unlikelihood of a return to the liberalism of the past as long as the rich continue to benefit from the decline in liberal values.
  • b)
    The way the debate around liberalism has been captured by the rich who have managed to insulate themselves from economic hardships.
  • c)
    The hypocrisy of the liberal rich, who profess to subscribe to liberal values while cornering most of the wealth.
  • d)
    The fact that the rise in liberalism had led to a greater interest in shared futures from unlikely social classes.
Correct answer is option 'C'. Can you explain this answer?

EduRev GMAT answered
  • Option C is the correct answer because the mention of "the Davos elite" in the passage serves to illustrate the perceived hypocrisy of wealthy individuals who profess to adhere to liberal values while simultaneously amassing the majority of the wealth. The author points to the contradiction between the elite's participation in events like discussions on creating a shared future and their exclusive access to privileges, symbolized by the use of private planes. This highlights the critique that the liberal rich, represented by the Davos elite, may not align their actions with the egalitarian ideals they claim to support, emphasizing a discrepancy between rhetoric and behavior.
  • Option A is incorrect because the passage does not explicitly connect the decline in liberal values to the rich benefiting, but rather to internal contradictions and hubris.
  • Option B is incorrect as the passage does not focus on how the debate around liberalism is captured by the rich; instead, it critiques the actions of the Davos elite.
  • Option D is incorrect because the passage does not suggest that the rise in liberalism has led to greater interest in shared futures from unlikely social classes; rather, it critiques the behavior of the Davos elite as hypocritical.

The passage below is accompanied by four questions. Based on the passage, choose the best answer for each question.
RESIDENTS of Lozère, a hilly department in southern France, recite complaints familiar to many rural corners of Europe. In remote hamlets and villages, with names such as Le Bacon and Le Bacon Vieux, mayors grumble about a lack of local schools, jobs, or phone and internet connections. Farmers of grazing animals add another concern: the return of wolves. Eradicated from France last century, the predators are gradually creeping back to more forests and hillsides. “The wolf must be taken in hand,” said an aspiring parliamentarian, Francis Palombi, when pressed by voters in an election campaign early this summer. Tourists enjoy visiting a wolf park in Lozère, but farmers fret over their livestock and their livelihoods. .
. .
As early as the ninth century, the royal office of the Luparii—wolf-catchers—was created in France to tackle the predators. Those official hunters (and others) completed their job in the 1930s, when the last wolf disappeared from the mainland. Active hunting and improved technology such as rifles in the 19th century, plus the use of poison such as strychnine later on, caused the population collapse. But in the early 1990s the animals reappeared. They crossed the Alps from Italy, upsetting sheep farmers on the French side of the border. Wolves have since spread to areas such as Lozère, delighting environmentalists, who see the predators’ presence as a sign of wider ecological health. Farmers, who say the wolves cause the deaths of thousands of sheep and other grazing animals, are less cheerful. They grumble that green activists and politically correct urban types have allowed the return of an old enemy.
Various factors explain the changes of the past few decades. Rural depopulation is part of the story. In Lozère, for example, farming and a once-flourishing mining industry supported a population of over 140,000 residents in the mid-19th century. Today the department has fewer than 80,000 people, many in its towns. As humans withdraw, forests are expanding. In France, between 1990 and 2015, forest cover increased by an average of 102,000 hectares each year, as more fields were given over to trees. Now, nearly one-third of mainland France is covered by woodland of some sort. The decline of hunting as a sport also means more forests fall quiet. In the mid-to-late 20th century over 2m hunters regularly spent winter weekends tramping in woodland, seeking boars, birds and other prey. Today the Fédération Nationale des Chasseurs, the national body, claims 1.1m people hold hunting licences, though the number of active hunters is probably lower. The mostly protected status of the wolf in Europe—hunting them is now forbidden, other than when occasional culls are sanctioned by the state—plus the efforts of NGOs to track and count the animals, also contribute to the recovery of wolf populations.
As the lupine population of Europe spreads westwards, with occasional reports of wolves seen closer to urban areas, expect to hear of more clashes between farmers and those who celebrate the predators’ return. Farmers’ losses are real, but are not the only economic story. Tourist venues, such as parks where wolves are kept and the animals’ spread is discussed, also generate income and jobs in rural areas.
The author presents a possible economic solution to an existing issue facing Lozère that takes into account the divergent and competing interests of:
  • a)
    politicians and farmers.
  • b)
    environmentalists and politicians.
  • c)
    farmers and environmentalists.
  • d)
    tourists and environmentalists.
Correct answer is option 'C'. Can you explain this answer?

EduRev GMAT answered
The passage mentions that farmers in Lozère are concerned about the return of wolves causing losses in livestock. On the other hand, environmentalists view the presence of wolves as a sign of wider ecological health. The suggested economic solution involves tourist venues related to wolves, such as parks, which not only address the economic concerns of farmers by generating income but also align with the interests of environmentalists who appreciate the return of the predators.
Therefore, Option C accurately captures the collaboration between farmers and environmentalists in the proposed solution.

The passage below is accompanied by four questions. Based on the passage, choose the best answer for each question.
The Second Hand September campaign, led by Oxfam . . . seeks to encourage shopping at local organisations and charities as alternatives to fast fashion brands such as Primark and Boohoo in the name of saving our planet. As innocent as mindless scrolling through online shops may seem, such consumers are unintentionally—or perhaps even knowingly —contributing to an industry that uses more energy than aviation. . . .
Brits buy more garments than any other country in Europe, so it comes as no shock that many of those clothes end up in UK landfills each year: 300,000 tonnes of them, to be exact. This waste of clothing is destructive to our planet, releasing greenhouse gasses as clothes are burnt as well as bleeding toxins and dyes into the surrounding soil and water. As ecologist Chelsea Rochman bluntly put it, “The mismanagement of our waste has even come back to haunt us on our dinner plate.”
It’s not surprising, then, that people are scrambling for a solution, the most common of which is second-hand shopping. Retailers selling consigned clothing are currently expanding at a rapid rate . . . If everyone bought just one used item in a year, it would save 449 million lbs of waste, equivalent to the weight of 1 million Polar bears. “Thrifting” has increasingly become a trendy practice. London is home to many second-hand, or more commonly coined ‘vintage’, shops across the city from Bayswater to Brixton.
So you’re cool and you care about the planet; you’ve killed two birds with one stone. But do people simply purchase a second-hand item, flash it on Instagram with #vintage and call it a day without considering whether what they are doing is actually effective?
According to a study commissioned by Patagonia, for instance, older clothes shed more microfibres. These can end up in our rivers and seas after just one wash due to the worn material, thus contributing to microfibre pollution. To break it down, the amount of microfibres released by laundering 100,000 fleece jackets is equivalent to as many as 11,900 plastic grocery bags, and up to 40 per cent of that ends up in our oceans. . . . So where does this leave second-hand consumers? [They would be well advised to buy] high-quality items that shed less and last longer [as this] combats both microfibre pollution and excess garments ending up in landfills. . . .
Luxury brands would rather not circulate their latest season stock around the globe to be sold at a cheaper price, which is why companies like ThredUP, a US fashion resale marketplace, have not yet caught on in the UK. There will always be a market for consignment but there is also a whole generation of people who have been taught that only buying new products is the norm; second-hand luxury goods are not in their psyche. Ben Whitaker, director at Liquidation Firm B-Stock, told Prospect that unless recycling becomes cost-effective and filters into mass production, with the right technology to partner it, “high-end retailers would rather put brand before sustainability.”
According to the author, companies like ThredUP have not caught on in the UK for all of the following reasons EXCEPT that:
  • a)
    Recycling is currently not financially attractive for luxury brands.
  • b)
    Luxury brands want to maintain their brand image.
  • c)
    Luxury brands do not like their product to be devalued.
  • d)
    The British don’t buy second-hand clothing.
Correct answer is option 'D'. Can you explain this answer?

EduRev GMAT answered
Option D is the correct answer because the passage does not mention or suggest that the British don't buy second-hand clothing. Instead, the passage discusses challenges related to luxury brands and their reluctance to circulate their latest season stock globally at a cheaper price. The reasons mentioned include the financial aspect(Option A), concerns about brand image(Option B), and the desire to avoid devaluing their products(Option D). Therefore, the passage does not attribute the slow adoption of companies like ThredUP in the UK to the British not buying second-hand clothing.

Chapter doubts & questions for Reading Comprehension - IBPS PO Prelims & Mains Preparation 2024 is part of Bank Exams exam preparation. The chapters have been prepared according to the Bank Exams exam syllabus. The Chapter doubts & questions, notes, tests & MCQs are made for Bank Exams 2024 Exam. Find important definitions, questions, notes, meanings, examples, exercises, MCQs and online tests here.

Chapter doubts & questions of Reading Comprehension - IBPS PO Prelims & Mains Preparation in English & Hindi are available as part of Bank Exams exam. Download more important topics, notes, lectures and mock test series for Bank Exams Exam by signing up for free.

Top Courses Bank Exams

Signup to see your scores go up within 7 days!

Study with 1000+ FREE Docs, Videos & Tests
10M+ students study on EduRev